Sunteți pe pagina 1din 119

CORRECTION OF ERRORS

1. Stratton Company has determined its 2014 and 2015 net income figures to be P1,500,000
and P1,100,000, respectively. In a first time audit of the company’s financial statements, you
determined the following errors:

a) Merchandise inventory was incorrectly determined: P50,000 overstatement for 2014


and P150,000 overstatement for 2012.
b) Revenue received in advance in 2014 of P250,000 was credited to a revenue
account when received. Of the total. P50,000 was earned in 2014, P120,000 was
earned in 2015 and the remainder will be earned in 2013.
c) P120,000 gain on sale of plant assets in 2015 was erroneously credited to
Accumulated Profits and Losses.

What is the corrected net income for the year 2015? P1,240,000

Net income for 2015 P1,100,000


a) Overstatement of 2014 inventory 50,000
Overstatement of 2015 inventory ( 150,000)
b) Understatement of revenue for 2015 120,000
c) Gain on sale-2015 not recognized in net income `120,000
Correct net income P1,240,000

2. Vinn Company reported an Accumulated Profits and Losses balabce of P300,000 at


December 31,2014. In June 2015, Vinn discovered that merchandise costing P100,000 had
not been included in inventory in its 2014 financial statements. Assume Vinn has 32% tax
rate.

What amount should Vinn report as adjusted beginning Accumulated Profits and Losses
on January 1, 2015? P368,000

Accumulated profits, December 31, 2014,


or January 1, 2015 P300,000
Understatement in inventory for 2014 P100,000
x Net of tax rate x 68% 68,000
Adjusted accumulated profits, January 1, 2015 P368,000

3. Casper Company reported an Accumulated Profits balance of P400,000 at December


31,2014. In August 2015, Casper Company determined that insurance premiums of P75,000
for the three-year period beginning January 1, 2014, had been paid and fully expensed in
2014. Assume Casper has a 32% income tax rate.

What amount should Casper report as adjusted beginning Accumulated Profits in


2015? P434,000

Accumulated profits, December 31, 2014 P400,000


Overstatement of expense P50,000
x Net of tax rate x 68% 34,000
Adjusted accumulated profits, January 1, 2015 P434,000

4. Oakman Company started operations on January 1, 2011. Financial statements for 2014 and
2015 contained the following errors:

December 31, 2014 December 31, 2015


Ending inventory P55,000 too high P65,000 too low
Depreciation expense 35,000 too high -
Insurance expense 25,000 too low 25,000 too high
Prepaid insurance 25,000 too high -

Additionally, a fully depreciated equipment was sold for P12,000 on December 31, 2015. The
sale was not recorded until 2016. No corrections have been made for any of the errors. (Ignore
income tax considerations)

How much would be the total effect of the errors in Oakman’s 2015 net income?
Understated by P157,000

2014 2015
Ending inventory:
2014 too high (P55,000) P 55,000
2015 too low 65,000
Depreciation-2014 too high 35,000
Insurance expense-2014 too low ( 25,000) 25,000
Unrecorded gain ________ 12,000
Effect on net income (P45,000) P157,000

How much would be the understatement in Oakman’s Accumulated Profits balance at


December 31, 2015? P112,000

Effect on 2014 net income (P 45,000)


Effect on 2015 net income 157,000
Net effect on December 31, 2015 Accumulated Profits P 112,000

5. On December 30,2014, Mazu Corporation sold merchandise for P75,000 to Nhoreen


Company. The terms of the sale were n/30, FOB shipping point. The merchandise was
shipped on December 31, 2014, and arrived at Nhoreen Company on January 2, 2015 and
the merchandise, sold at 25% markup on coast, was included in Mazu’s inventory at
December 31, 2014.

As a result, Mazu cost of goods sold for the year ended December 31, 2014 was?
Understated by P60,000

The December 31, 2014 inventory was overstated. Therefore, cost of goods sold for
2014 was understated bu P60,000 (P75,000 / 125%)
6. Star Company’s December 31 year-end financial statements had the following errors:

December 31, 2014 December 31, 2015


Ending inventory P13,500 understated P19,800 overstated
Depreciation expense 3,600 understated -
Unearned rental 5,000 understated -
Prepaid insurance 8,000 understated

There was no other errors during the years 2014 or 2015 and no corrections have been made
for any of the errors. (Ignore income tax consideration)
What is the net effect of the errors on Star’s 2015 net income? Overstated by P20,300

Effect on
Net Income December 31, 2015
2014 2015 Accumulated Profits
Ending inventory:
2014-understated P13,500 (P13,500) P 0
2015-overstated - ( 19,800) ( 19,800)
Depreciation-2014 under ( 3,600) - ( 3,600)
Unearned rental-2014 under ( 5,000) 5,000 -
Prepaid insurance-2015 under _______ 8,000 8,000_
Net effect (over) understated P 4,900 (P20,300) (P15,400)

What is the net effect of the errors in Star’s December 31, 2015 accumulated profits
balance? Understated by P8,000

What is the net effect of the errors in Star’s December 31, 2015 working capital?
Overstated by P11,800

Effect on Working
Capital (for 2015)
2015 overstatement of inventory (P19,800)
2015 understatement of prepaid insurance 8,000
Net effect on the working capital (over) (P11,800)

7. Records showed that as of December 31, 2014, accrued salaries payable of P21,000 were not
recorded in Sebby Company’s books. In addition, office supplies on hand of P9,000 at
December 31, 2015 were erroneously treated as expense instead of supplies inventory. Neither
of these errors was discovered nor corrected.

What is the effect of these two errors? 2015 net income is understated by P30,000 and
January 1, 2016 accumulated profits is understated by P9,000.

Accumulated
Net Income Profits
2014 2015 January 1, 2016
Unrecorded accrued salaries in 2014 (P21,000) P21,000 P 0
Office supplies on hand in 2015 charged
to expense ________ 9,000 9,000
Net effect – (overstated) understated (P21,000) P30,000 P9,000

8. Isay Corporation started operations on January 1, 2014. Financial statements for the years
ended December 31, 2014 and 2015 contained the following errors:

2014 2015
Ending inventory P240,000 understated P225,000 overstated
Depreciation expense 90,000 understated
Insurance expense 150,000 overstated 150,000 understated
Prepaid insurance 150,000 understated

Additionally, a fully depreciated equipment was sold for cash of P162,000 on December 31,
2015. The sale was not recorded until 2016. There were no other errors during 2014 or 2015
and no corrections have been made for any of the errors.

What is the total effect of the errors in the amount of the working capital at December 31,
2015? (ignore income taxes) Overstated by P63,000

Working Capital
2014 2015 Dec. 31, 2015
Inventory-2014 under P240,000 (P240,000)
Inventory-2015 over ( 225,000) (P225,000)
Depreciation-2014 under ( 90,000) 0 0
Prepaid insurance under 150,000 ( 150,000) 0
Gain on sale of equipment ________ 162,000 162,000
Net correction P300,000 (P453,000) (P 63,000)

9. While examining the accounts of Yo Mama Company on December 31, 2015, the following
errors were uncovered:

a) Dividends of P100,000 had been declared on December 15, 2015 but was not recorded
in the books.
b) Improvements in buildings and equipment for P480,000 had been debited to expense at
the end of April in 2014. Improvements are estimated to have an estimated life of 8
years.
c) The company failed to record sales commissions payable amounting to P10,500 and
P19,000 at the end of 2014 and 2015, respectively.
d) Supplies on hand amounting to P6,000 and P15,000 were not recognized at the end of
2014 and 2015, respectively.

What is the net effect of the above errors in the 2014 net income? P435,500 under
What is the neteffect of the error in the 2015 net income? P59,500 over
a) Unrecorded dividends No effect No effect
b) Leasehold improvements charged
to expense P480,000 under
understatement of depreciation
2014 (P480,000 / 8 x 8/12) 40,000 over
2015 (P480,000 / 8) P60,000 over
c) Unrecorded sales commissions
2014 10,500 over 10,500 under
2015 19,000 over
d) Unrecorded supplies on hand
2014 6,000 under 6,000 over
2015 _____________ 15,000 under
Net effect P435,500 under P59,500 over

10. The following errors were discovered in the course of examination of the Doodle Company’s
financial records:

o Year 2014 wages payable for P34,000 was not recorded.


o Accrued vacation pay for the year 2014 for P62,500 was not recorded because the
bookkeeper “never learned that you had to do it”.
o Insurance for a 12-month period purchased on November 1, 2014 was charged to
expense in the amount of P37,200 because “the amount of the check is about the same
every year”.

What is the net effect of the above errors on the January 1, 2015 accumulated profits?
P65,500 over

Accumulated Profits
Jan. 1, 2015
Under (Over)
Unrecorded wages payable (P34,000)
Accrued vacation pay for 2014 not recorded ( 62,500)
Overcharging of insurance expense during 2014
(P37,200 x 10/12) 31,000
Net effect on the 1/1/15 Accumulated Profits (P65,500)

11. Porpol Company discovered the following errors in its financial records at the beginning of the
year 2015:

a) The physical inventory count on December 31, 2014 excluded a merchandise with a cost
of P38,000 that had been temporarily stored in a public warehouse. Porpol uses the
periodic inventory system.
b) During 2014, a competitor filed a patent infringement suit against Porpol claiming
damages of P440,000. The company’s legal counsel hasindicated that an unfavorable
verdict is probable and a reasonable estimate of the court’s award to the competitor is
P250,000. The company has not reflected or disclosed this situation in the financial
statements.
c) A trademark was acquired at the beginning of 2013 for P100,000. No amortization has
been recorded since acquisition. It is the company’s policy to amortize all intangibles
with a definite life for a maximum of 20 years. At the time of acquisition, the trademark
was estimated to have a definite life of 20 years.

What is the effect of the above errors on the January 1, 2015 accumulated profits?
P222,000 overstated

Accumulated Profits,
January 1, 2015
Under (Over)
a) Company’s inventory, excluded in the physical count P 38,000
b) Failure to recognize a probable & reasonable amount
of estimated loss (250,000)
c) Failure to amortize trademarks (P100,000 / 20 x 2) ( 10,000)
Net effect (P222,000)

12. While examining the December 31, 2014 financial statements of Sunrise Company, you
discovered the following:

a) Inventory at January 1, 2014 had been overstated by P30,000


b) Inventory at December 31, 2014 was understated by P50,000
c) During 2014, Sunrise received a P100,000 cash advance from a customer for
merchandise to be manufactured and shipped during 2014. The amount was credited to
sales revenue.
d) The net income reported on the 2014 profit or loss before reflecting any adjustments for
the above items is P3,000,000

What is the corrected net income for the year ended December 31, 2014? P2,980,000

Reported net income P3,000,000


a) Overstatement of 1/1/14 inventory 30,000
b) Understatement of 12/31/14 inventory 50,000
c) Sales for 2015 recognized in 2014 ( 100,000)
Correct net income, 2014 P2,980,000

13. Bonjoy Corporation failed to recognize accruals and prepayments since the inception of its
business three years ago. The accruals and prepayments at the end of 2014 are given below:

Prepaid insurance P60,000


Accrued wages 75,000
Rent revenue collected in advance 96,000
Interest receivables 81,000

What is the net effect of the above errors in the 2014 net income? P30,000 overstated

Effect on Net Income


Understated (Overstated)
Understatement of prepaid insurance P 60,000
Understatement of accrued wages ( 75,000)
Understatement of rent revenue collected in advance ( 96,000)
Understatement of interest receivables 81,000
Overstatement of net income (P30,000)

14. Maan Company had the following financial statement information:

2012 2011
Revenue 1,350,000 1,000,000
Expenses 980,000 650,000
Net income 370,000 350,000

12/31/2012 12/31/2011
Total assets 1,570,000 1,050,000
Total liabilities 500,000 350,000
Total owner’s equity 1,070,000 700,000

Maan failed to record P120,000 of accrued wages at the end of 2011. The wages were recorded
and paid in January 2012. The correct accruals were made on December 31, 2012.

What is the corrected net income for 2011? 230,000

Net income for 2011 350,000


Unrecorded accrued wages – December 31, 2011 (120,000)
Corrected net income for 2011 230,000

15. What is the corrected net income for 2012? 490,000

Net income for 2012 370,000


Accrued wages on 12/31/2011 recorded in 2012 120,000
Corrected in net income for 2012 490,000

16. What is the correct amount of total liabilities on December 31, 2011? 470,000

Total liabilities – December 31, 2011 350,000


Unrecorded accrued wages – December 31, 2011 120,000
Correct amount of total liabilities – December 31, 2011 470,000

17. What is the correct amount of owner’s equity on December 31, 2012? 1,070,000

Total owner’s equity – December 31, 2012 1,070,000

18. Erich Company manufactures kerosene heaters for home use. The December 31 financial
statements contained the following error:
2010 2011
Ending inventory 200,000 under 300,000 over
Depreciation 50,000 under

An insurance premium of P150,000 was prepaid in 2010 to cover 2010, 2011 and 2012. The
entire amount was charged to expense in 2010. On December 31, 2011, fully depreciated
machinery was sold for P250,000 cash but the sale was not recorded until 2012. There were no
other errors during 2010 and 2011 and no corrections have been made for any of the errors.

Ignoring income tax, what is the net effect of the errors on the retained earnings on
December 31, 2011? 50,000 overstated

2010 ending inventory under 200,000 (200,000)


2011 ending inventory over (300,000)
2010 depreciation under ( 50,000) ---
Insurance premium 100,000 ( 50,000)
Gain on sale of machinery --- _ 250,000_
Net correction to income 250,000 (300,000)

Net correction to 2010 net income 250,000


Net correction to 2011 net income (300,000)
Net correction to retained earnings ( 50,000)

19. Bayle Company is in the process of adjusting its books at the end of 2011. Bayle’s records
revealed the following information:

 Bayle failed to accrue sales commissions at the end of 200 and 2010 as follows:

2009 220,000
2010 140,000

In each case, the sales commissions were paid and expensed in January of the
following year.

 Errors in ending inventory for the last three years were discovered to be as
follows:

2009 400,000 understated


2010 540,000 overstated
2011 150,000 understated

The unadjusted retained earnings balance on January 1, 2011 is P12,600,000 and the
unadjusted net income for 2011 was P3,000,000. Dividends of P1,750,000 were declared during
2011.

What is the adjusted net income for 2011? 3,830,000


2009 2010 2011
Unrecorded commissions:
2009 (220,000) 220,000
2010 (140,000) 140,000
Ending inventory:
2009 under 400,000 (400,000)
2010 over (540,000) 540,000
2011 under _______ ________ 150,000
Net correction to income 180,000 (860,000) 830,000

Net income per book for 2011 3,000,000


Net correction to income of 2011 830,000
Adjusted net income of 2011 3,830,000

20. What is the adjusted balance of retained earnings on December 31, 2011? 14,000,000

Net correction to income of 2009 180,000


Net correction to income of 2010 ( 860,000)
Net correction to income of prior years ( 680,000)

Retained earnings – January 1, 2011 12,600,000


Prior period errors ( 680,000)
Corrected beginning balance 11,920,000
Net income for 2011 3,830,000
Dividends declared in 2011 ( 1,750,000)
Retained earnings – December 31, 2011 14,000,000

ACCRUAL & CASH BASIS OF ACCOUNTING

1. Yord Company reported revenue of P6,000,000 under the cash basis for the year ended 2014.
Additional information was made available:

Accounts receivable, December 31, 2013 P1,250,000


Accounts receivable, December 31, 2014 1,375,000

Under the accrual basis, how much should Yord Company report as revenue for 2014?
P6,125,000

Cash basis revenue P6,000,000


Accounts receivable, 12/31/2014 1,375,000
Accounts receivable, 12/31/2013 ( 1,250,000)
Accrual basis revenue P6,125,000
2. Dream Company reported total purchases of P2,500,000 in its cash basis financial statement on
December 31, 2014. Additional information revealed the following:

Accounts payable, January 1, 2014 P 600,000


Accounts payable, December 31, 2014 800,000

Under the accrued basis of measuring revenues and expenses, how much is the total
purchases for the year ended December 31, 2014? P2,700,000

Purchases – Cash basis P2,500,000


Accounts payable, 12/31/2014 800,000
Accounts payable, 01/01/2014 600,000
Purchases – Accrual basis P2,700,000

3. Elsa Company’s professional fees expense account had a balance of P164,000 on December
31, 2014 before considering year-end adjustments relating to the following:

Consultants were hired for a special project at a total fee not to exceed P130,000. Elsa has
recorded P110,000 of this fee based on billings for work performed in 2014

The attorney’s letter requested by the auditors dated January 30, 2014 indicated that legal fees
of P12,000 were billed on January 15, 2015 for work performed in November 2014 and unbilled
fees for December 2014 were P14,000.

What amount of professional fees expense should Elsa report for the year ended
December 31, 2014 profit or loss? P190,000

Professional fees per book P164,000


Accrued legal fees –
November P12,000
December 14,000 24,000
Adjusted professional fees P190,000

4. In 2009, Leon Designs Corporation sold a layout design to Laica Inc. and will receive royalties of
20% of future revenues associated with the said layout design. On December 31, 2013, Leon
Designs reported royalties receivables of P75,000 from Laica Inc. during 2014, Leon Designs
received royalty payments of P200,000. Laica Inc. reported revenues of P1,500,000 in 2014
from the layout design.

In its 2014 profit or loss, what amount should Leon Designs report as royalty revenue?
P300,000

Reported revenue P1,500,000


x royalty rate x 20%
Royalty revenue P 300,000

5. To maintain sufficient operating cash, Infinity Company frequently borrows from a bank. Below
is the summary of loans granted to Infinity with 12% interest rate. The principal and the related
interest are payable at maturity and Infinity was able to repay the loans on scheduled maturity
date:

Date of Loan Amount Maturity Date Term of Loan


November 1, 2013 P300,000 October 31, 2014 1 year
February 1, 2014 900,000 July 31, 2014 6 months
May 1, 2014 480,000 January 31, 2015 9 months

Infinity records interest expense when the loans are repaid. Accordingly, interest expense of
P90,000 was recorded in 2014.

If no correction is made, by what amount would 2014 interest expense be understated?


P32,400

November loan (P300,000 x 12% x 10/12) P 30,000


February loan (P900,000 x 12% x 6/12) 54,000
May loan (P480,000 x 12% x 8/12) 38,000
Interest expense for 2014 P 122,400
Less: Interest recorded 90,000
Understatement of interest expense P 32,400

6. Polly Company owns an office building and leases the offices under a variety of rental
agreements involving rent paid in advance monthly or annually. Not all tenants make timely
payments of their rent. The following data were taken from the balance sheets of Polly
Company:

Rentals receivable were P96,000 and P124,000 for 2013 and 2014, respectively;
Unearned rentals were P320,000 and P240,000 for 2013 and 2014, respectively

During 2014, Polly received P800,000 cash from tenants.

What amount of rental should Polly record for 2014? P908,000

Collection of rentals P800,000


Rental receivables, 2014 124,000
Rental receivable, 2013 ( 96,000)
Unearned rentals, 2013 320,000
Unearned rentals, 2014 ( 240,000)
Rental revenue P 908,000

7. Anne Company reported revenue of P3,100,000 in its accrual basis income statement for the
year ended December 31. 2014. Additional information were as follows:

Accounts receivable, December 31, 2013 P 700,000


Accounts receivable, December 31, 2014 1,100,000

Under the cash basis, how much should Anne report as revenue for 2014? P2,700,000

Revenue – Accrual basis P3,100,000


Accounts receivable, 2013 700,000
Accounts receivable, 2014 ( 1,100,000)
Revenue – Cash Basis P 2,700,000

8. Sol Company reported revenue of P1,980,000 in its income statement for the year ended
December 31, 2014. Additional information was made available:

December 31, 2013 December 31, 2014


Accounts receivable P415,000 P550,000
Allowance for doubtful accounts 25,000 40,000

No uncollectible accounts were written off during 2014. Had the cash basis of accounting been
used instead, how much would have been reported as receipts for 2014 by Sol Company?
P1,845,000

Revenue – accrual P1,980,000


Accounts receivable, December 31, 2013 415,000
Accounts receivable, December 31, 2014 ( 550,000)
Collections/Receipts P1,845,000

9. Chief Company reported total purchases of P3,200,000 in its accrual basis financial statement
on December 31, 2014. Additional information revealed the following:

Accounts payable, December 31, 2013 P 900,000


Accounts payable, December 31, 2014 1,250,000

What is the amount of purchases under the cash basis on December 31, 2014?
P2,850,000

Purchases, accrual P3,200,000


Accounts payable, 12/31/2013 900,000 P4,100,000
Less: Accounts payables, 12/31/2014 1,250,000
Purchases, cash basis (payment of A/P) P2,850,000

10. Under the accrual basis, rental income of Pido Company for the calendar year 2014 is
P600,000. Additional information regarding rental income are presented below:

Unearned rental income, January 1, 2014 P50,000


Unearned rental income, December 31, 2014 75,000
Accrued rental income, January 1, 2014 30,000
Accrued rental income, December 31, 2014 40,000

Under the cah basis, how much rental income should be reported by Pido Company in the
year 2014? P615,000

Revenue – Accrual basis P600,000


Accrued rental income, January 1, 2014 30,000
Unearned rental income, December 31, 2014 75,000
Accrued rental income, December 31, 2014 ( 40,000)
Unearned rental income, January 1, 2014 ( 50,000)
Rental income – Cash basis P615,000

11. Red Boys Corporation acquires copyright from authors, paying advance royalties in some cases
and in others, paying royalties within 30 days of year-end. Red Boys reported royalty expense of
P375,000 for the year ended December 31, 2014. The following data are included in the
corporation’s December 31 balance sheet:

2013 2014
Prepaid royalties P60,000 P50,000
Royalties payable 75,000 90,000

Under the cash basis, what amount of royalty expense should report in its 2014 profit or
loss? P350,000

Royalties expense – Accrual basis P375,000


Royalties payable, 2013 75,000
Prepaid royalties, 2014 50,000
Royalties payable, 2014 ( 90,000)
Prepaid royalties, 2013 ( 60,000)
Royalty expense – Cash basis P350,000

12. Burn Corporation maintains its accounting records on the cash basis but restates its financial
statements to the accrual method of accounting. Burn had P600,000 in cash-basis pretax
income for 2014. The following information pertains to Burn’s operations for the years ended
December 31, 2014 and 2013:

2014 2013
Accounts receivable P400,000 P200,000
Accounts payable 150,000 300,000

Under the accrual method, what amount of income before taxes should Burn report in its
December 31, 2014 profit or loss? P950,000

Net income, Cash basis P600,000


Increase in accounts receivable 200,000
Decrease in accounts payable 150,000
Net income, accrual basis P950,000

13. At December 31, 2014, the advertising expense account of Apo Company had a balance of
P146,000 before any year-end adjustment relating to the following:

 Brochures and leaflets for a sales promotional campaign in January 2015


amounting to P15,000 was included in the P146,000 balance.
 Airtime for the radio advertisements during December 2014 for P9,000 was billed
to Apo on January 2, 2015. Apo paid the full amount on January 9, 2015.
What amount should Apo report as advertising expense in its profit or loss for the year
ended December 31, 2014? P140,000

Unadjusted balance P146,000


Prepaid advertising ( 15,000)
Accrued advertising – December 9,000
Advertising expense P140,000

14. Greg, a lawyer, maintains his accounting records under the cash basis of accounting. During
2014, Greg collected P200,000 in fees from clients. At December 31, 2014, Greg has accounts
receivable of P40,000. At December 31, 2014, Greg had accounts receivable of P60,000 and
unearned fees of P5,000.

On accrual basis, what was Greg’s service revenue for 2014? P215,000

Collection P200,000
Accounts receivable, December 31, 2014 60,000
Accounts receivable, December 31, 2013 ( 40,000)
Unearned fees, December 31, 2014 ( 5,000)
Service revenue, accrual P215,000

15. Icon Publishers offered a contest in which the winner would receive P1,000,000 payable over 20
years. On December 31, 2014, Icon announced the winner of the contest and signed a note
payable to the winner for P1,000,000, payable in P50,000 installments every January 2. Also,
on December 31, 2014, Icon purchased an annuity for P418,250 to provide the P950,000 prize
monies remaining after the first P50,000 installment, which was paid on January 2, 2014.

In its 2014 profit or loss, what should Icon report as contest prize expense? P468,250

1st installment of notes payable due 2014 P 50,000


Cost of annuity purchases on December 31, 2014 418,000
Contest prize expense for 2014 P468,250

16. Ara started operating a service proprietorship on April 1, 2014 with an initial cash investment of
P120,000. The business provided P38,400 of services in April and received full payment in May.
The business incurred expenses of P18,000 in April which were paid in June. During May, Ara
drew P6,000 against his capital account.

What was the income for the two months ended May 31, 2014 under the following method
of accounting? Cash Basis – P38,400; Accrual Basis – P20,400

Revenue P38,400 P38,400


Expense 0 ( 18,000)
Net income P38,400 P20,400
17. Spike Corporation pays commissions to its sales agents at the rate of 3% of net sales. Sales
agents are not paid salaries but are given monthly advances of P15,000. Advances are
changed to commission expense, and reconciliation against commissions are prepared
quarterly. Net sales for the year ended March 31, 2014 were P15,000,000. The unadjusted
balance in the commissions expense account on March 31, 2014 was P400,000. March
advances were paid on April 3, 2014.

In its profit or loss for the year ended March 31, 2014, what amount should Spike report
as commission expense? P450,000

Sales P15,000,000
x Commission rate x 3%
Commission expense P 450,000

18. On September 1, 2014, Marie began a service proprietorship with an initial investment of
P400,000. Marie provided P800,000 of services during September. Collections were made
except for P200,000 which were paid the following month. Expenses were incurred in the
amount of P400,000, including P100,000 which are to be paid next month. Marie withdrew
P60,000 against the capital account.

In September 30,2014 financial statement, what amount of capital should be reported


under the cash basis accounting? P640,000

Capital, beginning P400,000


Add: Net income – Cash Basis* 300,000
Total P700,000
Less: Withdrawals ( 60,000)
Capital balances, September 30, 2014 P640,000

Revenue – cash basis (P800,000 – P200,000) P600,000


Expenses – cash basis (P400,000 – P100,000) 300,000
*Net income P300,000

19. During 2011, Kaye Company had P200,000 in cash sales and P3,000,000 in credit sales. The
account receivable balances were P4,00,000 and P485,000 at December 31, 2010 and 2011,
respectively. If Kaye desires to prepare a cash basis income statement, what amount should
be reported as sales for 2011? 3,115,000

Accounts receivable – December 31, 2011 400,000


Credit sales 3,000,000
Total 3,400,000
Less: Accounts receivable – December 31, 2011 485,000
Collections 2,915,000
Cash sales 200,000
Total sales – cash basis 3,115,000
20. REED Company, which began operations on January 1, 2010, has elected to use cash basis
accounting for tax purposes and accrual basis accounting for its financial statements. Reed
reported sales of P1,750,000 and P800,000 in its tax returns for the years ended December 31,
2011 and 2010, respectively. Reed reported accounts receivable of P300,000 and P500,000 on
December 31, 2022 and 2010, respectively.

What amount should Reed report as sales in its income statement for 2011? 1,550,000

Accounts receivable – December 31, 2011 300,000


Add: Sales in 2011 under cash basis 1,750,000
Total 2,050,000
Less: Accounts receivable – December 31, 2010 500,000
Sales – accrual basis 1,550,000

JOB ORDER COSTING

1. Under Khayla Company’s job order costing system, manufacturing overhead is applied to work-
in-process using a predetermined annual overhead rate. During January 2013, Khayla’s
transactions included the following:

Direct materials issued to production P 90,000


Indirect materials issued to production 8,000
Manufacturing overhead incurred 125,000
Manufacturing overhead applied 113,000
Direct labor costs 107,000

Khayla had neither beginning nor ending work-in-process inventory. What was the cost of jobs
completed in January 2013? P310,000

Direct materials issued to production P 90,000


Direct labor costs 107,000
Applied factory overhead 113,000
P310,000

2. Bal Corporation manufactures rattan furniture sets for export and uses the job order cost system
in accounting for its costs. You obtained from the corporation’s books and records the following
information for the year ended December 31, 2013:

 The work in process inventory on January 1 was 20% less than the work in process
inventory on December 31.
 The total manufacturing costs added during 2011 was P900,000 based on actual direct
materials and direct labor but with manufacturing overhead applied on actual direct labor
pesos.
 The manufacturing overhead applied to process was 72% of the direct labor pesos, and
it was equal to 25% of the total manufacturing costs.
 The cost of goods manufactured, also based on actual direct materials, actual direct
labor and applied manufacturing overhead was P850,000.

The cost of direct materials used and the work-in-process inventory on December 31,
2013: P362,500

Total manufacturing cost P900,000


Less: applied factory overhead (25% x P900,000) 225,000
P675,000
Prime costs
Less: direct labor costs
Applied factory overhead P225,000
Divided by: % of direct labor costs 72% 312,500
Direct materials used P362,500

3. A company allocates overhead to jobs in process using direct labor costs, raw material costs,
and machine hours. The overhead application rates for the current year are:

100% of direct labor


20% of raw materials
P117 per machine hours

A particular production run incurred the following costs:


Direct labor, P8,000
Raw materials, P2,000
A total of 140 machine hours were required for the production run

What is the total cost that would be charged to the production run? P24,780

100% x P8,000 of direct labor = P 8,000


20% x P2,000 of raw materials = 400
P117 x 140 machine hours = 16,380
P24,780

4. Crown, Inc had the following information relating to 2013.

Budgeted factory overhead P74,800


Actual factory overhead 78,300
Applied factory overhead 76,500
Estimated labor hours 44,000

If Crown decides to use the actual results from 2013 to determine the 2014 overhead rate, what
will be the 2013 overhead rate be? P1,740
The 2013 overhead rate is calculated as P1.70 / DH (P74,800 , 44,000 DLH). Since
applied factory overhead is a result of actual DLH times the overhead rate, the actual direct
labor hours for 2013 are 45,000 (P76,500 , P1.70). next, the overhead rate for 2014 is P1.74 /
DLH (P78,300 + 45,000 DLH)

5. The PDF Company uses a predetermined overhead rate. PDF prepared the following budget at
the beginning of the year:

Direct labor cost P12.000


Factory overhead 25,000
Direct labor hours 9,000
Machine hours 1,500

During the month of January, the cost sheet of order number 100 indicates P20 of raw
materials, P50 of direct labor, 10 hours of direct labor, and 5 machine hours. Order number 100
consists of 49 units of product. PDF applies overhead based on direct labor cost. What amount
of overhead should be applied to order number 100? 104.7

PDF Company applies overhead based on direct labor cost. Since overhead was
budgeted at P25,000 and direct labor cost at P12,000, the overhead application rate is P2.083
(P25,000 , P12,000). Since 50 of direct labor was incurred, P104.17 of overhead should be
applied (50 x P2.083). alternatively, factory overhead cost is slightly more than twice the direct
labor cost (P25,000 / P12,000) and the overhead on a job with 50 of direct labor cost would be
slightly more than P100, i.e., P104.17

6. ABC Corporation is a manufacturing company engaged in the production of a single special


product known as “Marvel”. Production costs are accumulated with the use of a job0order-cost
system.

The following information is available as of June 1, 2014:

Work-in-process P10,710
Direct materials inventory 48,600

In analyzing the job-order cost sheets, the records disclosed that the composition of the work-in-
process inventory on June 1, 2014 were as follows:

Direct materials used P 3,960


Direct labor (900 hours) 4,500
Factory overhead applied 2,250
P10,710

The following manufacturing activity occurred during the month of June 2014:

Purchased direct materials costing P60,000


Direct labor worked 9,900 hours at P5 per hour
Factory overhead of P2.50 per direct labor hour was applied to production
At the end of June 2014, the following information was gathered in connection with the
inventories:
Inventory of work-in-process:
Direct materials used P12,960
Direct labor (1,500 hours) 7,500
Factory overhead applied 3,750
P24,210
Inventory of direct materials P51,000

What is the cost of goods manufactured? P118,350

Direct materials inventory, June 1, 2014 P 48,600


Add: Purchases 60,000
Direct materials available for use P 108,600
Less: Direct materials inventory, June 30, 2014 51,000
Direct materials used P 57,600
Direct labor (9,900 hours x P5/hour) 49,500
Applied factory overhead (9,900 hours x P2.5/hour) 24,750
Manufacturing cost P 131,850
Add: Work-in-process, June 1, 2014 10,710
Total work placed in process P 142,560
Less: Work-in-process, June 30, 2014 24,210
Cost of goods manufactured P 118,350

7. Steady Corporation’s materials purchase during 2014 are P25,590 and materials put into
production are direct and indirect materials, respectively, worth P18,500 and P7,090. The total
factory payroll is P74,000 of which P50,000 represents direct labor. Other factory overhead
costs amount to P32,000. The company applies the actual factory overhead cost to process.
Sales, cost of goods sold, and the cost of goods manufactured, respectively, are P130,000,
P120,000, and P128,000.

By what amount did the company’s closing goods in process inventory exceed its
opening goods in process inventory? P3,590

Direct materials P 18,500


Direct labor 50,000
Actual factory overhead:
Indirect Materials P 164,250
Indirect Labor (P74,000 – P50,000) 225,525
Other factory overhead costs 32,000 63,090
Manufacturing cost P131,590
Less: cost of goods manufactured 128,000
Increase in foods-in-process P 3,590

Increase in goods in process indicates that the closing goods in process is higher than
the opening goods in process. Thus, increase means zero beginning to P3,590 ending.

8. Dan company consumed P450,000 worth of direct materials during May 2014. At the end of the
month, the direct materials inventory was P25,000 lower than the May 1 inventory level. How
much was the direct materials procured during May 2014? P425,000
Direct materials used P 450,000
Less: decrease in inventory 25,000
Direct materials purchased P 425,000

9. Job No. 027 has, at the end of the second week in April, an accumulated total cost of P4,200. In
the third week, P1,010 of direct materials were used on the Job.

Twenty (20) hours of direct labor services were applied to the job at a cost of P5 per hour.

Manufacturing overhead was applied at the basis of P2.50 per direct labor hour for fixed
overhead and P2 per hour for variable overhead.

Job No. 027 was only the job completed during the third week.

What is the total cost of Job Order No. 027? P5,400

Work-in-process, beginning P4,200


Added:
Direct materials P1,010
Direct labor (20 hours x P5) 100
Applied factory overhead (20 hours x P4.5) 90 1,200
Total cost of Job 027 P5,400

10. The XYZ Corporation manufactures one product and accounts for cost by a job-order cost
system. You have obtained the following information for the year ended December 31, 2013
from the corporation’s books and records:

Total manufacturing cost added during 2013


based on actual direct materials, actual
direct labor and applied factory overhead
on actual direct labor cost P1,000,000

Cost of goods manufactured based on actual


direct materials and direct labor and
applied factory overhead 970,000

Applied factory overhead to work in process


based on direct labor cost 75%

Applied factory overhead for the year, based on


total manufacturing cost 2%

Beginning work in process inventory was 80% of ending work in process inventory.

Compute the cost of direct materials used for the year ended December 31, 2013.
P370,000

Direct materials used + Direct labor + Overhead = Manufacturing Cost


Direct materials used + Direct labor + (P1,000,000 x 27%) = P1,000,000
DM, used + OH / 75% + P270,000 = P1,000,000
DM, used + P270,000 / 75% + P270,000 = P1,000,000
DM, used + P360,000 + P270,000 = P1,000,000
DM, used = P1,000,000 – P630,000
DM, used = P370,000

11. Red Company incurred the following costs during the month: direct labor, P120,000; factory
overhead, P108,000; and direct materials purchases, P160,000. Inventories show the following
costs:

Beginning Ending
Finished goods P27,000 P30,000
Work in process 61,500 57,500
Direct materials 37,500 43,500

How much is the cost of goods manufactured? P386,000

Direct materials, beginning P 37,500


Add: Direct materials purchases 160,000
Direct materials available for use P 197,500
Less: direct materials, ending 43,500
Direct materials used P 154,000
Direct labor 120,000
Factory overhead 108,000
Manufacturing cost P 382,000
Add: Work-in-process, beginning 61,500
Total work-placed in process P 443,500
Less: Work-in-process, ending 57,500
Cost of goods manufactured P 386,000

12. Nikkon Company, Ic. Estimated its factory overhead at P510,000 for the year, based on a
normal capacity of 100,000 direct labor hours. Standard direct labor hours for the year totaled
105,000, while the factory overhead control account at the end of the year showed a balance of
P540,000. How much was the underapplied factory overhead for the year? P4,500
Underapplied overhead is the difference between the actual overhead and the applied
overhead. The overhead rate per hour is P5.10 (P510,000 budgeted costs, 100,000 budgeted
hours). The applied overhead was P535,500 (P5.10 x 105,000 actual hours). The actual
overhead incurred of P540,000 less the applied overhead of P535,500 results in P4,500 of
underapplied overhead.

13. Wesley Company has underapplied overhead of P45,000 for the year 2014. Before disposition
of the underapplied overhead, selected year-end balances from Wesley’s accounting records
were:

Sales P1,200,000
Cost of goods sold 720,000
Direct materials inventory 36,000
Work-in-process inventory 54,000
Finished goods inventory 90,000

Under Wesley’s cost accounting system, over-or underapplied overapplied is allocated to


appropriate inventories and CGS based on year-end balances in its year-end income statement,
What amount should Wesley report of CGS? P757,500

The allocation of underapplied overhead increases CGS. The underapplied overhead of


P45,000 for the year should be allocated on a pro rata basis to work-in-process (P54,000),
finished goods (P90,000), and CGS (P720,000). The sum of these three items is P864,000.
Thus, P37,500 should be allocated to CGS [(P720,000 / 864,000) x P45,000]. CGS after
allocation is P757,500 (P37,500 + P720,000). The remaining P7,500 should be allocated
proportionately between work-in-process and finished goods.

COST of QUALITY: SPOILAGE AND DEFECTIVE UNITS

14. Kho Company manufactures electric drills to the exacting specifications of various customers.
During April 2014, Job 143 for the production of 1,100 drills was completed at the following
costs per unit:

Direct materials P10


Direct labor 8
Applied factory overhead 12
TOTAL P30

Final inspection of Job 143 disclosed 50 defective units and 100 spoiled units. The defective
drills were reworked at a total cost of P500, and the spoiled drills were sold to a jobber for
P1,500. What would be the unit cost of the good units produced on Job 143? P32

The original production of 1,100 drills cost P33,000 (1,100 drills x P30 per drill). The
reworking of the defective drills (i.e. P500) increased the cost total to P33,500. The P1,500
received from the sale of the 100 defective units should be subtracted from the total cost
incurred in producing the 1,100 drills. Therefore, the total cost for producing 1,000 good drills
equals P32,000 (P33,000 + P500 – P1,500), yielding a unit cost for good drills of P32.

15. Daisy Manufacturing Corporation started 150 units in process on Job Order No.007. The prime
costs placed in process consisted of P30,000 and P18,000 for materials and direct labor,
respectively, and a pre-determined rate was used to charge factory overhead to production at
133-1/3% of the direct labor cost. Upon completion of the job order, units equal to 20% of the
good output were rejected for failing to meet strict quality control requirements.

The company sells rejected units as scrap at only 1/3 of production cost, and bills customers at
150% of production cost.

If the rejected units were ascribed to company failure, the billing price of Job Order No.
0007 would be? P90,000
Units %
Started in process 150 120%
Less: Spoilage ( 25) 20%
Goods output 125 100%

Charged to Work-in-process:
Materials P30,000
Labor 18,000
Overhead (P18,000 x 133 1/3%) 24,000 P72,000
Less: Spoilage Cost (P72,000 , 150 = P480 x 25 units) 12,000
Net Cost of Production P60,000
Multiplied by: Billing Price 150%
Billing Price P90,000

16. Using the same information in No. 14, and if the rejected units were ascribed to customer
action, the billing price of Job Order No. 007 would be? P102,000

Charged to work-in-process P 72,000


Less: Spoilage cost at salvage value (1/3 x P12,000) 4,000
Net cost of production P 68,000
Multiplied by: Billing Price 150%
Billing price P102,000

17. Hailey Corporation’s Job 123 for the manufacture of 2,200 coats, which was completed during
August at the unit costs presented below. Final inspection of Job 123 disclosed 200 spoiled
coats which were sold to a jobber for P6,000.

Direct materials P20


Direct labor 18
Factory overhead (includes an allowances
of P1 for spoiled work) 18
P56

Assume that spoilage loss is charged to all production during August. What would be the unit
cost of the good coats produced on Job 123? P 56 /unit

The unit cost of goods produced includes direct materials, direct labor, and factory
overhead. Since the spoilage is included in the calculation of overhead, it must be considered
normal and a product cost. Thus, the unit cost remains at P56 since the amount to be deducted
from the production cost is equivalent to the original cost per unit. The computation is as follows:

Charged to work-in-process:
P56 x 2,200 P123,200
Less: Spoilage cost: P56 x 200 11,200
Net cost of production P112,000
Divided by: Number of good units produced 2,000
P 56/unit
18. Using the same information in #16, assume instead that the spoilage loss is attributable to the
exacting specifications of Job 123 and is charged to the specific job. What would be the unit
cost of the good coats produced on Job 123? P57.50/unit

If the spoilage is charged to this specific job (rather than to factory overhead), the
spoilage allowance should be removed from the factory overhead rate. The overhead
application rate thus drops to P17 because this job’s spoilage is not typical and will not be
averaged with other “typical” jobs. The costs of producing the 2,000 good coats include the
costs incurred in the production of the 2,200 coats, less the P6,000 received for the spoiled
coats. The unit costs is net cost of production divided by the number of good coats produced
(2,000).

2,000 (P20 + P18 + P17) – P6,000


------------------------------------------------ = P57.50/unit
2,000

SERVICE COST ALLOCATION

19. A company has two service departments (S1 and S2) and two production departments (P1 and
P2). Department data for January were as follows:

S1 S2
Costs incurred: P27,000 P18,000
Service provided to:
S1 - 20%
S2 10% -
P1 50% 30%
P2 40% 50%

What are the total allocated service department costs to P2 if the company uses the
reciprocal method of allocating its service department cost? (Round calculation to the
nearest whole number) 23,051

The reciprocal method allocates service department costs to other service departments
as well as to production departments by means of simultaneous equations, as shown below.
Thus, total service cost allocated to P2 is P23,051 [(40% x P31,224) + (50% x P21,122)].

S1 = P27,000 + .2S2
= P27,000 + [2(P18,000 + 1S1)]
= P27,000 + P3,600 + .02S1
.98S1 = P30,600
S1 = P31,224

S2 = P18,000 + /1(P31.224)
= P18,000 + P3,122
S2 = P21,122
20. A hospital has a P100,000 expected utility bill this year. The janitorial, accounting, and orderlies
department are services functions to the operating, hospital rooms and laboratories
departments. Floor space assigned to each department is

Department Square Footage

Janitorial 1,000
Accounting 2,000
Orderlies 7,000
Operating 4,000
Hospital Rooms 30,000
Laboratories 6,000
50,000

How much of the P100,000 will eventually become the hospital rooms department total
costs, assuming a direct allocation boased on square footage is used? P75,000

The direct method allocates service department costs without regard to service provided
to other service department. consequently, the P100,000 utility expense will be apportioned
among the three production departments on the basis of square footage. The hospital rooms
department’s share will be P75,000 (P100,000 x [30,000 square feet / (4,000 + 30,000 + 6,000)
total square feet]).

PROCESS COSTING

1. Borge Company computed the flow of physical units completed for Department M for the month
of March 2011 as follows

Units completed:

From work-in-process on March 2011 15,000


From March production 45,000
60,000

Material are added at the beginning of the process. The 12,000 units of work in process at
March 31, 2011, were on 80% complete as to convertion costs. The work-in-process at March 1,
2011 was 60% complete as to conversion costs.

Using the FIFO method, the equivalent units for March conversion costs were: 60,600

Work EP-
Quantity Schedule: Actual Done CC
In process, beginning …………... 15,000
Started in process ……………… 57,000
72,000

Accounted for as follows:


In process, beginning, F&T ……. 15,000 40% 6,000
Started, F&T ……………………... 45,000 100% 45,000
In process, ending ……………… 12,000 80% 9,600
72,000 60,600

2. Cost and statistics for Department 2 at a company manufacturing a single product in three
department follows:

Work-in-process, October 1:
Cost in Department 1…………………………….. P11,380
Cost in Department 2
Materials…………………………………….. None
Labor………………………………………… P500
Factory overhead………………………….. 50

Cost in Department 2 in October:


Materials……………………………………………. None
Labor……………………………………………….. P13,000
Factory overhead…………………………………. 450
Unit in process, October 1, 60% completed
As to conversion costs………………………….. 500
Units received from Department 1 in October at P2.60 per unit 6,700
Units completed and transferred to Department 3 in October 6,800
Units in process, October 31, half completed
As to conversion costs………………………….. 400

Compute the conversion costs per equivalent unit (rounded to nearest centavo)
FIFO: P2.01 Average: P2.00

Work EP-
Quantity Schedule Actual Done CC
In progress, beginning ………….. 500
Received from Prec. Dept …….... 6,700
7,200

Accounted for as follows – FIFO


In progress, beg., F&T ………….. 500 40% 200
Received, F&T (6,800 – 500) ….. 6,300 100% 6,300
In process, ending ........................ 400 50% 200
7,200 6,700
Accounted for as follows – Average
Finished and transferred ………. 6,800 100% 6,800
In process, ending …………….... 400 50% 200
7,200 6,700

Conversion Costs per Equivalent Unit:


FIFO (P13,000 + P450) / 6,700 ………………………………….. P2.01

Average: (P500 + P50 + P13,000 + P450) / 7,000 ……………. P2.00

3. Earl Company uses process costs system with average costing to account for the production of
its only product. The product is manufactured in two departments. Units are started then
transferred to the Finishing Department, where they are completed. Units are inspected at the
end of the production process in the Forming Department, and the costs of abnormal lost is
charged to Factory Overhead Control account or abnormal spoilage expense depending on
management prerogative. Data related to August operations in the Forming Department are:

Units in the beginning invemtory (60% materials,


35% labor, and 25% overhead)……………….. 1,000
Units started in process this period………………..…. 9,000
Units transferred to Finishing Department
This period…………………………………...…….. 8,000
Units in ending inventory (100% materials,
75% labor, and 50% overhead)………………. 1,500

Costs charged to the department: Beginning Added


Inventory this period y
Materials………………….. P1,260 P36,240
Direct Labor……………… 770 10,780
Factory overhead……….. 1,400 21,725

Total costs transferred to Finishing Department: P59,600

Work EP- Work EP- Work EP-


Quantity Schedule: Actual Done Mat. Done Lab. Done CC
In process, beginning … 1,000
Started in process …….. 9,000

10,000

Accounted for as follows:


Finished and transferred 8.000 100% 8,000 100% 8,000 100% 8,000
In process, end ………… 1,500 100% 1,500 75% 1,125 50% 750
Abnormal lost ………….. 500 100% 500 100% 500 100% 500
10,000 10,000 9,625 9,250

Total Cost Transferred:


Materials: (P1,260 + P36,240) / 10,000 ……………………………………….. P3.75
Direct Labor: (P770 + P10,780) / 9,625 ……………………………………….. 1.20
Overhead: (P1,400 + P21,725) / 9,250 ………………………………………... 2.50
Cost per equivalent unit …………………………………………………………. P7.45
Multiplied by: No. of units transferred …………………………………………. 8,000
P59,600
4. Roy Company manufactures Product x in a two-stage production cycle in Department A and B.
materials are added in the beginning process as to the 6,000 units in BWIP and 75% complete
as to the 8,000 units in EWIP, 12,000 units were completed and transferred out of Department B
during February. An analysis of the costs relating to WIP and production activity in Department
B for February follows:

Transferred Materials Conversion


in Costs Costs Costs
WIP, Fenruary 1:
Costs attached P12,000 P2,500 P1,000
Feb. activity:
Costs added P29,000 P5,000 P5,000

The total costs per unit transferred-out for February of Product x, rounded to the
Nearest peso: P2.78

Work EP- Work EP-


Quantity Schedule: Actual Done Mat. Done CC
In process, beginning ………….. 6,000
Received from Prec. Dept. 14,000
20,000

Accounted for as follows – FIFO


In process, beg. , F&T ………….. 6,000 - - 50% 3,000
Received, F&T (12,000 – 6,000) 6,000 100% 6,000 100% 6,000
In process, ending ……………… 8,000 100% 8,000 75% 6,000
20,000 14,000 15,000

Accounted for as follows – Average


Finished and transferred ……… 12,000 100% 12,000 100% 12,000
In process, ending ……………... 8,000 100% 8,000 75% 6,000
20,000 20,000 18,000

Materials – 100%
l------------------------------------------------------------------------------------------------------------------------------l
0 50% 75% 100%

EUP: Mat. – 0; CC – 50%


X-----------------------------------X
IP, beg.

IP, end EUP: Mat. – 100%; CC – 75%


X--------------------------------------------------------------------X

FIFO:
In process beg. , F&T
Cost last month: (P12,000 + P2,500
+ 1,000) ………………………………… P15,500
Cost this month:
Materials ……………………………….. -0-
CC: 3,000 x (P5,000 / 15,000) ……..... 1,000 P16,500
Received, F&T: 6,000 x [(P29,000 / 14,000)
+ (P5,500 / 14,000) + (P5,000 / 15,000)] ………………… 16,785
Total Cost transferred ……………………………………………… P33,285
Divided by: F&T ……………………………………………………... . 12,000
Unit Cost transferred ……………………………………………….. P2.77

Average:
Preceding Department: (P12,000 + 29,000) / 20,000 ………….. P2.05
Current/This Department:
Materials (P2,500 + P5,500) / 20,000 ……………………. .40
CC: (P1,000 + P5,000) / 18,000 …………………………... .33
P2.78

5. Department Z of the Cobra Mfg. Corporation had the following data for the month of October,
2011:

Beginning work in process, 70% complete…………… 40,000 units


Started process during the month……………………... 300,000 units
Ending work in process, 80% complete………………. 60,000 units

The costs of the beginning work in process was P140,000, and the production costs for
The month amounted to P1,172,000.

How many equivalent production units were completed in October, 2011 using FIFO
Method? 300,000

Work Equivalent
Quantity Schedule: Actual Done Production

In process, beginning ………………. 40,000


Started in process …………………… 300,000
340,000

Accounted for as follows:


In process, beginning, F&T ………… 40,000 30% 12,000
Started, F&T …………………………. 240,000 100% 240,000
In process, ending …………………... 60,000 80% 48,000
340,000 300,000

6. Grace co., a manufacturer of combs, budgeted sales of 125,000 units for the ,on the of April.
The following additional information is provided:

Number
of units
Actual inventory at April 1
Work-in-process………………………………………… None
Finished goods…………………………………………. 37,500
Budgeted inventory at April 30
Work-in-process (70% processed) ………………….. 8,000
Finished goods ………………………………………… 30,000

How many equivalent units of production did Grace budget for April? 123,500

Given that there is no beginning work-in-process, it makes no difference whether FIFO


or Average Method is used. The equivalent works of units for April would be as follows

Budgeted Sales ………………………………………………………… 125,000


Add: Finished goods, April 30 ………………………………………... 30,000
Units available to be sold ……………………………………………... 155,000
Less: Finished goods, April 1 ………………………………………… 37,500
Units completed ………………………………………………………… 117,500
Add: Evuivalent units of production – work-in
Process, April 30 (8,000 x 75%) …………………………………… 6,000
Total uints placed in process …………………………………………. 123,500
Less: EUP- work in process, April 1 ……………………………………….. ______-
Total Expected Equivalent Units of Prod. (EUP) ………………………… 123,500

7. Kirin Corporation’s production cycle starts in the Mixing Department. The following information is
available for April:

Units
WIP, April (50% complete) …………………………………… 40,000
Started in April ………………………………………………… 240,000
WIP, April 30 (60% complete) ………………………………. 25,000

Materials are added at 55% stage of completion in the Mixing Department. What are the
Equivalent units of production for the month of April?

FIFO Average
Materials Conversion Materials Cpnversion
280,000 250,000 280,000 270,000

Work EP- Work EP-


Quantity Schedule: Actual Done Mat. Done CC
In process, beg. ………………….. 40,000
Started-in process ……………….. 240,000
280,000

Accounted for as follows – FIFO


In process, beg. , F&T …………… 40,000 100% 40,000 50% 20,000
Started, F&T (280,000 –
40,000 – 25,000) …………… 215,000 100% 215,000 100% 215,000
IP, ending ………………………… 25,000 100% 25,000 60% 15,000
In process, ending ………………. 280,000 28,000 250,000

Materials – 100%
I
I------------------------------------------------------------------------------------------------------------------------------I
0 50% 55% 60% 100%
IP, beg. EUP: M – 100%; CC – 50%
X----------------------------------------------------------------X
IP, end.
EUP: M – 100% ; CC – 60%
X------------------------------------------------------------------------------------------X

Accounted for as follows – Average


Finished and transferred ……… 255,000 100% 255,000 100% 255,000
In process, ending ……………... 25,000 100% 25,000 60% 15,000
280,000 280,000 270,000

8. Department A is the first stage of Brick Company’s production cycle. The following information is
available for conversion costs for the month of April 2011:

Units
Work-in-process, beginning (60% complete) ………………. 20,000
Started in April ………………………………………………….. 340,000
Completed in April and transferred to Department B …….. 320,000
Work-in-process, ending (40% complete) ………………….. 40,000

The equivalent units for the conversion cost calculation are: FIFO Average
324,000 336,000

Work Conversion
Quantity Schedule: Actual Done Cost
In progress, beg. ………… 20,000
Started in process ………. 340,000
360,000

Accounted for as follows – FIFO


In process, beg. , F&T ………….. 20,000 40% 8,000
Started F&T (320,000 – 20,000) 300,000 100% 300,000
In process, ending ……………… 40,000 40% 16,000
360,000 324,000

Accounted for as follows – Average


Finished and transferred ……… 320,000 100% 320,000
In process, ending ……………... 40,000 40% 16,000
360,000 336,000

9. Bark Company manufactures compact disks. In June 2011, production for 2,000,000 units were
started. At the end of the month, the following data were gathered:
Completed units ……………………………………………………. 2,700,000
Defective units ……………………………………………………… 400,000
In process ½ complete ……………………………………………. 800,000

How many units were in process at the beginning of the month? 1,900,000

Quantity Schedule:
In progress, beg. (balancing figure) ………………………………. 1,900,000
Started in process ……………………………………………………. 2,000,000
3,900,000

Accounted for as follows:


Completed units ……………………………………………………… 2,700,000
In process, ending ………………………………………………….... 800,000
Defective units ………………………………………………………… 400,000
3,900,000

10. The following production information for Dept. B of Joy Products is for the month of May, 2011:

Received from Dept. A ………………………………….. 600,000 units


Completed and transferred to Dept. C ………………... 500,000 units

Additional information:
a. No beginning work process
b. Ending work in process is 75% complete
c. May’s production costs total P2,760,000

Dept. B’s unit cost of production for May, 2011 is: P4.80

Work Equivalent
Quantity schedule: Actual Done Production
Received from Department A … 600,000

Accounted for as follows:


Finished and transferred ………. 500,000 100% 500,00
In process, ending ……………… 100,00 0 75% 75,000\
600,000 575,000

Cost per Equivalent Unit: P2,760,000 / 575,000 ………………………………. P4.80

11. Gayson Company, which had 6,000 units n work-in-process at January 1 that were 60%
complete as to conversion costs. During January 20,000 units were completed. At January 31,
8,000 units remained in WIP which were 40% complete as to conversion costs. Materials are
added at the beginning of the process.

Using the weighted average method, the equivalent units for January for conversion
costs were: 23,200

Work EP –
Quantity Schedule: Actual Done CC
In process, beg. ……………………. 6,000
Started in process (28,000 – 6,000) 22,000
28,000

Accounted for as follows:


Finished and transferred …………. 20,000 100% 20,000
In process, ending ……………….... 8,000 40% 3,200
28,000 23,000

12. The Norton Company manufactures the famous ticktock watch on an assembly line basis.
January 1, work-in-process consisted of 5,000 units partially completed. During the month an
additional 110,000 units were started and 105,000units were completed, the ending work-in-
process wa 3/5 complete as to conversion cost. Conversion costs are added evenly throughout
the process. The following conversion costs were incurred.

Beginning costs for work-in-process …………………………… P1,500


Total current conversion costs ………………………………….. 273,000

The conversion costs assigned to ending work-in-process totaled P15,360 using the
FIFO method of process costing. What was the percentage of completion, as to
conversion costs on the 5,000 units in BWIP? 80%

Ending work in process is 10,000 (5,000 + 110,00 – 105,000) units which are 3/5 complete
representing 6,000 equivalent units of conversion (10,000 x 3/5). Conversion cost per unit is
P15,360 / 6,000 units which is P2.56 per unit. Total equivalent conversion units for the month is
P273,920 / P2.56/unit = 107,000 units. Total equivalent conversion units are the sum of
equivalent units to finish beginning units, units started and completed, and ending equivalent
units. Assuming that 100,000 units were started this period and completed, the amount of
conversion added to BWIP this period is determent below:

107.000 units = BWIP + units started and completed + EWIP


107,000 units = BWIP + 100,000 + 6,000 units
BWIP = 1,000 units this period

That means BWIP was equal to 4,000 EUP with respect to conversion, or 80% complete (4,000
+ 5,000).

13. During March, Mark Company’s Department Y equivalent unit product costs, computed under
the weighted average method, were as followed:

Materials ……………………………………………………………. P1
Conversion …………………………………………………………. 3
Transferred-in ………………………………………………………. 5

Materials are introduced at the end of the process in Department Y. there were 4,000
Units (40% complete as to conversion costs) in WIP at March 31. The total costs
Assigned to the March 31 WIP inventory should be P24,800

The unit cost of EUP under weighted average are already computed. EWIP consists of 4,000
units 40% complete as to conversion costs (1,600 EUP). Since materials are added at the end
of the process, there is no material cost, only transferred-in cost.
Cost of in-process, March 31:
Cost from preceding dept.: P5 x 400 …………………………………… P20,000
Cost this dept.:
Materials: P1 x 0 ………………………………………………….. 0
CC: P3 x (4,000 x 40%) …………………………………………. 4,800
P24,800

14. Barkley company adds materials at the beginning of the process in Department N. Data
concerning the materials used in March 2011 production are as follows:

Units
Work-in-process at March 1 …………………………………….. 16,000
Started during March …………………………………………….. 34,000
Completed and transferred to next department during March... 36,000
Normal spoilage incurred ……………………………………….. 4,000
Work-in-process at March ………………………………………. 10,000

The equivalent units for the materials unit cost calculation are:
FIFO Average
30,000 46,000

Work EP –
Quantity Schedule: Actual Done Mat.
In process, beginning …………………… 16,000
Started in process ……………………….. 34,000
50,000

Accounted for as follows – FIFO


In process, beg. , F&T …………………… 16,000 - -
Started, F&T (36,000 – 16,000) ………... 20,000 100% 20,000
In process, ending ………………………. 10,000 100% 10,000
Normal lost ………………………………. 4,000 - -
50,000 30,000
Accounted for as follows – Average
Finished and transferred ………………. 36,000 100% 36,000
In process, ending …………………….... 10,000 100% 10,000
Normal lost ……………………………… 4,000 - -
50,000 46,000

15. Materials are added at the start of the process in Ceasar Company’s blending department, the
first stage of the production cycle. The following information is available for July:

Units
Work-in-process, July 1 (60% complete
As to conversion costs) ………………………………. 60,000
Started in July …………………………………………………. 150,000
Transferred to the next department …………………………. 110,000
Lost in production …………………………………………….. 30,000
Work-in-process, J?uly 31 (50% complete
As to conversion costs) …………………………….. 70,000

Under Ceasar’s cost accounting system, the costs incurred on the lost units are
Absorbed by the remaining good units. What are the equivalent units for the
Materials unit cost calculation? FIFO Average
120,000 180,000

Work EP –
Quantity Schedule: Actual Done Mat.
In process, beginning …………………… 60,000
Started in process ……………………….. 150,000
210,000

Accounted for as follows – FIFO


In process, beg. , F&T …………………… 60,000 - -
Started, F&T (110,000 – 60,000) ……..... 50,000 100% 50,000
In process, ending ………………………. 70,000 100% 70,000
Normal lost ……………………………….. 30,000 - -
210,000 120,000

Accounted for as follows – Average


Finished and transferred ……………….. 110,000 100% 110,000
In-process, ending ………………………. 70,000 100% 70,000
Normal lost ………………………………. 30,000 - -
210,000 180,000

16. Frank Co. had the following production for the month of June.

Units
Work-in-process at June 1 ……………………………………….. 10,000
Started during June ……………………………………………….. 40,000
Completed and transferred to finished goods during June…. 33,000
Abnormal spoilage incurred ……………………………………… 2,000
Work-in-process during June 30 ………………………………… 15,000

Materials are added of the beginning of the process. As to conversion cost, the
Beginning work-in-process was 70% complete and the ending work-in-process was
60% completed. Spoilage are detected at the end of the process. The equivalent units
For June, with respect to conversion costs, were as follows:
FIFO Average
37,000 44,000

Work EP –
Quantity Schedule: Actual Done CC
In process, beginning ………………….. 10,000
Started in process ……………………… 40,000
50,000

Accounted for as follows – FIFO


In process, beg. ………………………… 10,000 30% 3,000
Started, F&T …………………………….. 23,000 100% 23,000
In process, ending ……………………… 15,000 60% 9,000
Abnormal lost …………………………… 2,000 100% 2,000
50,000 37,000

Accounted for as follows – Average


Finished and transferred ………………. 33,000 100% 33,000
In process, ending ……………………… 15,000 60% 9,000
Abnormal lost ……………………………. 2,000 100% 2,000
50,000 44,000
Since abnormal lost happens at the end of the production the work done should be equivalent to
100% because they were determined to be lost when production is already completed.
17. On September 30, work-in-process totaled 9,000 units 60% complete (based on conversion
costs added uniformly throughout the department and the material added at the start of the
process). a total of 100,000 units were transferred to the next department during October 31, at
a total of 8,000 units 40% complete (based on conversion costs) were still in process in
Department A.

Using the weighted-average cost flow method, which of the following equivalent units
should be used in the calculating of costs for October?

Equivalent Units
Transfer Costs Materials Conversion
108,000 108,000 103,000
Work EP- Work EP-
Quantity Schedule: Actual Done Mat. Done CC
In process, beginning ………….. 9,000
Started in process ……………… 99,000
108,000

Quantity Schedule
Finished and transferred ………. 100,000 100% 100,000 100% 100,000
In process, ending …………….... 8,000 100% 8,000 40% 3,200
108,000 108,000 103,000

18. Information for the month of January concerning Department A, the first stage of Ogden
Corporation’s productive cycle, is as follows:

Materials Conversion
BWIP ……………………………………… P8,000 P6,000
Current costs…………………………….. 40,000 32,000
Total costs ………………………………. P48,000 P38,000

Equivalent units using weighted


Average method ……………….. 100,000 95,000
Average unit costs ……………………… P0. 48 P0.40

Goods completed ……………………… 90,000 units


EWIP …………………………………….. 10,000 units

Materials are added at the beginning of the process. the ending work-in-process is 50%
Complete as to conversion costs. How would the total costs accounted for be
distributed, Using the weighted average method?
Goods Ending Work-
Completed -In-Process
P79,200 P6,800

The weighted average method combines the costs in BWIP those for the current period.
Materials are added at the beginning of the process and conversion costs are assumed to be
incurred uniformly. Equivalent unit and average until cost calculations were given.

Completed goods:
Materials (P .48 x 90,000) …………………………………………………….. P43,200
Conversion costs (P. 40 x 90,000) ………………………………………….. 36,000
Cost of completed goods …………………………………………………………….. P79,200

Given that conversion costs for EWIP are 50% complete, there are 5,000 (50% x 10,000)
equivalent units of conversion cost in ending inventory.

Cost of in-process, ending:


Materials (P . 48 x 10,000) …………………………………………………... P4,800
Conversion costs (P .40 x 5,000) …………………………………………… 2,000

Cost of EWIP …………………………………………………………………………… P6,800

19. For the month of May, 2011, the Finishing Dept. of Apple, Inc. had in opening work in process
80% complete units and in ending work in process 50% complete units. Related data for the
month follow:
Units Conversion cost
Work in process, May 1 ………………………….. 50,000 P88,000
Units started, and costs incurred during May … 270,000 572,000
Units completed and transferred during May … 200,000

If the company uses first-in, first-out costing, the conversion cost of the work in process
at the end of May would be: P156,000

Work EP –
Quantity Schedule Actual Done CC
In process, beginning …………………… 50,000
Started in process ……………………….. 270,000
320,000

Accounted for as follows:


In process, beg. , F&T ………………........ 50,000 20% 10,000
Started, F&T (20,000 – 50,000) ………... 150,000 100% 150,000
In process, ending ………………………. 120,000 50% 60,000
320,000 220,000

Cost of In-process, end: (P572,000 / 220,000) x 60,000 ……………………… P156,000

20. Barnet Company adds materials at the beginning of the process in Department M. Conversion
costs were 75% complete as to the 8,000 units in WIP at May 1 and 50% complete as to the
6,000 units in WIP at May 31. During May 1, 12,000 units were completed and transferred to the
next production activity for May is as follows:

Costs
Materials Conversion
WIP, May 1 ……………………………….. P9,600 P4,800
Costs added in May …………………….. 15,600 14,400

The total cost per equivalent until for May was:


FIFO Average
3.16 2.68

Work EP- Work EP-


Quantity Schedule: Actual Done Mat. Done CC
In process, beginning …………… 8,000
Started in process ………………. 10,000
18,000

Accounted for as follows – FIFO


In process, beg. , F&T …………… 8,000 - - 25% 2,000
Started, F&T (12,000 – 8,000) …. 4,000 100% 4,000 100% 4,000
In process, ending ………………. 6,000 100% 6,000 50% 3,000
18,000 10,000 9,000

Accounted for as follows – Average


Finished and transferred ……….. 12,000 100% 12,000 100% 12,000
In process, ending ………………. 6,000 100% 6,000 50% 3,000
18,000 18,000 15,000

Cost per Equivalent Unit:


FIFO: Materials CC Total
Cost added during May …. P15,600 P14,400
Divided by: EUP ………….. 10,000 9,000
P1.56 P1.60 P3.16

Average:
In process, beginning …… P9,600 P4,800
Cost added during May … 15,600 14,400
Total ………………………. P25,200 P19,200
Divided by: EUP ………… 18,000 15,000 P2.68
JOINT AND BY-PRODUCTS

1. Horse Co. manufactures products A and B from a joint process. During October, 2011. Sales
values at the point of “split-off” were P50.000 for 4,000 units of product A and P100,000 for
12,000 units of product B. Selling prices per unit are P25.00 and P12.50, respectively, for
product A and for product B.

Assume that the joint cost allocated to product A by using the market value method was
P40,000. The production cost of product B would be reported at: P130,000

MV at Joint Furhter Process Total


Product Split-off pt. x % = Costs + Cost _ + Cost
A P50,000 80%* P40,000
B 100,000 80% 80,000 P500,00** P130,000
P150,000 P120,000
*P40,000 / P50,000 = 80%
**Ultimate Market value or final sales value: P25 x 4,000 …………….. P100,000
Less: Market Value at split-off point for Product A ……………………. 100,000
Further processing costs of Product B …………………………………. P50,000

2. Using the same information in No. 1, and assume that the joint cost allocated to product B by
using the average cost method was P90,000. The production cost of product A would be
reported at: P80,000

Units Joints Further Proc Total


Product Produced x Ave. U.C = Costs + Costs = Cost
A 4,000 P7.5 P30,000 P50,000** P80,000
B 12,000 7.5* 90,000
16,000 P120,000

*P90,000 , 12,000 = P7.5


**The Ultimate Market Value or Final Sales Value: P25 x 4,000 ……. P100,000
Less: Market Value at slpit-off point for Product A ………………….. 50,000
Further processing cost of Product A ……………………………………. P50,000

3. A company manufactures products X and Y using a joint process. the joint processing costs are
P10,000. Products X and Y can be sold at split-off for P12,000 and P8,000 respectively. After
spilt-off, product X is processed further at a cost of P5,000 and sold for P21,000 whereas
product Y is sold without further processing. If the company uses the net realizable value
method for allocating joint costs, the joint cost allocated to X is 6,000

Under the net realizable value method, joint costs are allocated based on their relative
net realizable value unless sales price quotations are available at split-off. Since split-off
sales price quotations are available, the amount of joint costs allocated to product X can
be computed as follows:
MV at %* Joint
Product Split-off Costs
X P12,000 50% P6,000
Y 8,000 50%
P20,000 P10,000
*P10,000 / P20,000

4. Crank Corporation, which manufactures two products out of a joint process – Compod and
Ultrasene. The joint (common) costs incurred are P250,000 for a standard production run that
generates 120,000 gallons of Compod and 80,000 gallons of Ultrasene. Compod sells for P2.00
per gallon while Ultrasene sells for P3.45 per gallon.

If there are no additional processing costs incurred after the split-off point, the amount of
joint cost of each production run allocated to Compod on a physical-quantity basis is
P150,000

Gallons
Product Produced x Ave. UC* = Joint Costs
Compod 120,000 P1.25 P150,000
Ultrasene 80,000 1.25 100,000
200,000 P250,000

*250,000 / 200,000 – P1.25 per gallon

5. Using the same information in No. 4, and if there are no additional processing costs
incurred after the split-off, the amount of joint cost of each production run allocated to
Ultrasene on a realizable value (gross market value) basis is 130,000

Gallons
Product Produced MV at SoPt. Total MV of SoPt. x %* = Joint Costs
Compod 120,000 P2,00 P240,000 50% P
Ultrasene 80.000 3.25 260,000 50% 130,000
P500,000 P250,000

*P250,000 / P500,000 = 50%


Since there is no further processing cost given, it is presumed that MV is at split-off point.

6. Using the information in No. 4 and if additional processing costs beyond the split-off point
are P.10 per gallon for Compod and P1.10 per gallon for Ultrasene, the amount of joint
cost of each production run allocated allocated to Ultrasene on a physical quantity basis
is 100,000

The additional processing costs are irrelevant if the allocation is based on physical
quantities the joint costs allocated to Ultrasene would be P100,000 (80,000 x P1.25 per gallon).

7. Using the information in No. 4 and if additional processing costs beyond the split-off point
are P.10 per gallon for Compod and P1.10 per gallon for Ultrasene, the amount of joint
cost of each production run allocated to Compod on a net realizable value (net market
value) basis is P180,000

Since, in this particular number, further processing costs were given, therefore, it is
presumed that the market value given is the ultimate or final sales price.

Gallons Ult. Or Final Further NRV or Total NRV


Product Produced Sales Price Proc. Cost/Gallons MV/Gallon or Hy. MV
Compod 120,000 P2.00 P.10 P1.90 P228,000
Ultrasene 80,000 3.25 1.10 2.15 172,000
400,000

Total NRV or Joint


Hyp. Market Value %* Costs
Compod P228,000 62.5% P142,500
Ultrasene 172,000 62.5%
P400,000 P250,000

*P250,000 / P400,000 = 62.5%

8. Earl Corporation, which manufactures a product that gives rise to a by-product called “Zafa”.
The only costs associated with Zafa are selling costs of P1 for each unit sold. Earl accounts for
Zafa sales first by deducting its separable costs from such sales. And then by deducting this net
amount from cost of sales of the major product. This year, 1,000 unit of Zafa were sold at P4
each.

If Earl changes its method of accounting for Zafa sales by showing the net amount as
additional sales revenue, Earl’s gross margin will Be unaffected

The gross margin equals sales minus cost of sales. Before the change, the net amount
was deducted from cost of sales. After the change, the net amount is added to regular sales
with no additional increase in cost of goods sold. Hence, the gross marginwill be the same.

9. Using the same information in No. 8, and Earl changes its method of accounting for Zafa
sales by showing the net amount as other income Earl’s gross margin will decrease by
P3,000

Sales revenue minus cost of goods sold is gross margin. If net revenue from the by-
product is recorded as other income rather than being deducted from cost of goods sold, the
gross margin will decrease by P3,000 [1,000 x (P4 sales – P1 cgs)].

10. Using the same information in No. 8, and Earl records the net realizable value for Zafa as
inventory as it is produced, what will the per unit value be? P3

The NRV is selling price minus cost to complete and cost to dispose. The selling price of
Zala is P4, and the selling costs are P1. Given no completion or additional processing costs,
unit net realizable value is P3.
11. Using the same information in No. 8, and Earl sold 1,000 units of Zafa. Assuming that 1,500
units were produced for the year and that net realizable value is recorded as inventory,
Earl’s net income will increase by: P4,500

If the 1,500 units of the by-product are recognized at the time of production, net income
must increase by P4,500 (P3 unit NRV x 1,500). Ending inventory of Zala reduces cost of sales,
and by-products revenue either decreases costs or increases other income

12. Using the same information in No. 8, and Earl records Zafa inventory at net realizable value
as it is produced this year, what will be the profit recognized next year on as sale of 500
unit? P0

Because NRV is selling price minus completion and disposal cost, there is no profit upon
sale. The sale of 500 units of Zala with an inventory value of P3 per unit will produce no profit
(P4 unit selling price 0 P3 inventory cost – P1 selling cost = P0)

13. The characteristic which is most often used to distinguish a product as either a joint
product or a by-product is the relative sales value of the products produced in the
process.

The difference between joint products and by-products lies in their relative sales values.
Joint products have relative sales values that are significant in relation to each other. By-
products have minor sales values compared with the major product(s).

14. Mace Co. manufactures a major product that gives rise to a by-product called May. May’s only
separable cost is a P1 selling cost when a unit is sold for P4. Mace accounts for May’s sales by
deducting the P3 net amount from the cost of goods sold at the major product. There are no
inventories. If Mace were to change its method of accounting for may from by-product to a
joint product, what would be the effect on Mace’s overall gross margin? Gross margin
increases by P1 for each unit of May sold.

The difference between treating the by-product “May” as a joint product against a by-
product would be under the by-product treatment, the selling cost is netted against “May’s”
selling price, thus, reducing gross profit whereas under joint product accounting, the selling
costs would be deducted below the gross profit like a selling expense.

15. Mann Sawmill manufactures two lumber products from a joint milling process. the two products
developed are mine support braces (MSB) and unseasoned commercial building lumber (CBL).
A standard production run incurs joint costs of P300,000 and results in 60,000 units of MSB and
90,000 units of CBL. Each MSB sells for P2 per unit, and each CBL sells for P4 per unit.

Assuming no further processing work is done after the split-off point, the amount of joint
cost allocated to commercial building lumber (CBL) on a physical quantity allocation
basis would be: P180,000
UNITS
PRODUCT PRODUCED X AVE. U.C. = JOINT COSTS

CBL 90,000 P2.00 P180,000


MSB 60,000 2.00 ___________
150,000 P300,000

16. Using the same information in No. 15, and there are no further processing costs incurred
after the spilt-off point, the amount of joint cost allocated to the mine support braces
(MSB) on a relative sales value basis would be: P75,000

UNITS MV. At TOTAL MV JOINT


PRODUCT PRODUCED X SOPT. = at SOPT. X %* = COSTS
CBL 90,000 P4 P360,000 62.5%
MSB 60,000 2 120,000 62.5% P75,000
P480,000 P300,000

17. Mugfield Corporation, which manufactures products C, D, and E from a joint process. joint costs
are allocated on the basis of relative sales value at split-off. Additional information is presented
below:

C D E Total
Units produced ………………… 6,000 4,000 2,000 12,000
Joint costs …………………........ P72,000 ? ? P120,000
Sales value at split off ………... ? ? P30,000 P200,000
Additional costs if processed
Further …………………. P14,000 P10,000 P6,000 P30,000
Sales value if processed further P140,000 P60,000 P40,000 P240,000

How much of the joint costs should Mugfield allocate to product D? P30,000

MV at SoPt. X %* = JOINT COSTS


C P120,000** 60% P 72,000
D 50,000 60% 30,000**
E 30,000 60% 18,000
P200,000 P120,000

*P 120,000 / P200,000 = 60%


** C = P72,000 / 60% = P120,000
** D = P200,000 – P120,000 – P30,000 – P50,000
** D’s share in joint costs would be: P50,000 x 60% = P30,000

18. Using the same information in No. 17, and assuming that the 2,000 units of product E were
processed further and sold for P40,000, what was Mugfield’s gross profit on the sale?
P16,000

Sales of Product E if process further P40,000


Less: Cost of Sales
Joint costs (refer No. 17) P 18,000
Further processing costs 6,000 24,000
Gross profit on sale of Product E P16,000

19. Which of the following components of production are allocable as joint costs when or
single manufacturing process produces several salable products? Materials, Labor and
Overhead

The difference between joint products and by-products lies in their relative sales value
compared to other products being produced. The term joint products is used when the relative
sales values of the individual products are significant in relation to each other. The term by-
products is used when the products hace a minor sales value as compared with the major
product(s).

20. Which of the following is often subject to further processing in order to be salable?
By-products – YES; Scrap – No

Scrap and by-products are usually similar in nature, both physically and from an
accounting point of view. Scrap might even be considered a by-product of a manufacturing
process. however, by-products generally have a greater sales value than scrap. And also, scrap
rarely receives any additional processing in order to be salable. The moment scrap are further
processed, then they are considered as by-products.

PARTNERSHIP

1. The following condensed balance sheet is presented for the partnership of AA, BB, and CC,
who share profits and losses in the ratio of 4:3:3, respectively:

Cash P 90,000
Other assets 830,000
AA, loan 20,000
P 940,000

Accounts payable P 210,000


CC, loan 30,000
AA, capital 310,000
BB, capital 200,000
CC, capital 190,000
P 940,000
Assume that the assets and liabilities are fairly valued on the balance sheet and that the
partnership decides to admit FF as a new partner, with a 20% interest. No goodwill or bonus is
to be recorded.

How much should FF contribute in cash or other assets? P175,000

Total agreed capital of the new partnership


(P310,000 + P200,000 + P190,000 / 80%) P 875,000
Less: Contribution of old partners (LL, PP, and QQ) ( 700,000)
Cash investment of FF P 175,000

or, alternately:
Total agreed capital of the new partnership P 875,000
Multiplied by: Capital interest of FF 20%
P 175,000

2. RR and PP share profits after the provision of annual salary allowances of P14,400 and
P13,200, respectively in the ratio of 6:4. However, if partnership’s net income is insufficient to
provide for said allowances in full amount, the net income shall be divided equally between the
partners. In 2014, the following errors were discovered: Depreciation for 2014 is understated by
P2,100 and the inventory on December 31, 2014 is overstated by P11,400. The partnership net
income for 2014 was reported to be P19,500.

The capital accounts of the partners should be increased (decreased) by:


RR, P(6,750); PP. P(6,750)

Correcting the allocated net income:


RR PP Total
Correct allocation of net income, equally P 3,000 P 3,000 P 6,000
Allocation of net income per books, equally 9,750 9,750 19,500
Adjustments increased (decreased) P(6,750) (P6,750) P13,000

3. Hunt, Rob, Turman, and Kelly own a publishing company that that operate as a partnership. The
partnership agreement includes the following:

 Hunt receives a salary of P20,000 and a bonus of 3% of income after all


bonuses.
 Rob receives a salary of P10,000 and a bonus of 2% of income after all bonuses.
 All partners are to receive 10% interest on their average capital balances.

The average capital balances are as follows:

Hunt P 50,000
Rob 45,000
Turman 20,000
Kelly 47,000

Any remaining profits and losses are to be divided equally among the partners.
Determine how a profit of P105,000 would be allocated among the partners.
Salaries P20,000 P10,000 - - P30,000
Bonus* 3,000 2,000 - - 5,000
10% Interest on Ave. Cap. 5,000 4,500 P 2,000 P 4,700 16,200
Balance (equally) 13,450 13,450 13,450 13,450 53,800
P41,450 P29,950 P15,450 P18,150 P105,000

*Bonus = (3% + 2%) (Net Income – Bonus)


B = 5% (P105,000 – B)
B = P5,250 - .05 B
1.05 B = P5,250
B = P5,250 / 1.05
B = P5,000, therefore Hunt should receive P3,000 (P5,000 x 3/5),
while Rob will receive P2,000 (P5,000 x 2/5)

4. AA and DD created a partnership to own and operate a health-food store. The partnership
agreement provided that AA receive a salary of P10,000 and DD a salary of P5,000 to
recognize their relative time spent in operating the store. Remaining profits and losses were
divided 60:40 to AA and DD, respectively. Income for 2013, the forst year of operations, of
P13,000 was allocated P8,000 to AA and P4,200 to DD.

On January 1, 2014, the partnership agreement was changed to reflect the fact that DD could
no longer devote any time to the store’s operations. The new agreement allows AA a salary of
P18,000, and the remaining profits and losses are divided equally. In 2014 an error was
discovered such that the 2013 reported income was understated by P4,000. The partnership
income of P25,000 for 2013 included the P4,000 related to year 2013.

In the reported net income of P25,000 for the year 2014, AA and DD would have:

AA BB CC
Salary P18,000 P18,000
Balance: Equally 1,500 P 1,500 3,000
Income for year 2014 only P19,500 P 1,500 P21,000
Income for year 2013 (60:40) 2,400 1,600 4,000
Reported income for year 2014 P21,900 P 3,100 P25,000

5. On April 30, 2014, XX, YY and ZZ formed a partnership by combining their separate business
proprietorship. XX contributed cash of P75,000. YY contributed property with a P54,000 carrying
amount, a P60,000 original cost, and P120,000 fair value. The partnership accepted
responsibility for the P52,500 mortgage attached to the property. ZZ contributed equipment with
a P45,000 carrying amount, a P112,500 original cost, and P82,500 fair value. The partnership
agreement specifies that profits and losses are to be shared equally but in silent regarding
capital contributions.

Which partner has the largest April 30, 2014, capital balance?

XX YY ZZ
Cash P75,000
Property P120,000
Equipment P82,500
Less: Mortgage assumed _______ 52,500 _______
Capital balances P75,000 P 67,500 P82,500

6. On December 1, 2013, EE and FF formed a partnership, agreeing to share for profits and losses
in the ratio of 2:3, respectively. EE invested a parcel of land that cost him P25,000. FF invested
P30,000 cash. The land was sold for P50,000 on the same date, three hours after formation of
the partnership.

How much should be the capital balance of EE right after formation? P50,000

In the formation of a partnership, one or more of the partner will contribute noncash assets to
the business such as inventory, land or equipment, etc.. Retaining the recorded cost for such
asset would be inequitable to any partners investing appreciated property. Therefore, the
contribution of noncash assets to a partnership should be recorded based on the fair values. In
this case, the fair value of the land would be measured by its sales price on the date of sale,
P50,000.

7. On August, AA and BB pooled their assets to form a partnership, with the form to take over their
business assets and assume the liabilities. Partners capitals are to be based on net assets
transferred after the following adjustments. (Profit and loss are allocated equally.)

BB’s inventory is to be increased by P4,000; an allowance for doubtful accounts of P1,000 and
P1,500 are to be set up in the books of AA and BB, respectively; and accounts payable of
P4,000 is to be recognized in AA’s books. The individual trial balances on August 1, before
adjustments are as follows:

AA BB
Assets P75,000 P113,000
Liabilities 5,000 34,500

What is the capital of AA and BB after the above adjustments?

Assets P 75,000 P113,000


Less: Liabilities 5,000 34,500
Unadjusted capital P 70,000 P 78,500
Add (Deduct): Adjustments;
Increase in inventory 4,000
Allowance for doubtful accounts ( 1,000) ( 1,500)
Accounts payable ( 4,000) ________
Adjusted capital balances P 65,000 P 81,000

8. The capital accounts of the partnership of NN, VV, and JJ on June 1, 2014 are presented below
with their respective profit and loss ratios:

NN P139,200 1/2
VV 208,800 1/3
JJ 96,000 1/6
On June 1, 2011, LL is admitted to the partnership when LL purchased, for P132,000, a
proportionate interest from NN and JJ in the net assets and profits of the partnership. As a result
of a transaction LL acquired a one-fifth interest in the net assets and profits of the firm.

What is the combined gain realized by NN and JJ upon the sale of a portion of their
interest in the partnership to LL? P43,200

Amount paid P132,000


Less: Book value of interest acquired:
(P139,200 + P208,800 + P96,000) x 1/5 88,800
Gain P 43,200

The problem is simpler than it appears at first glance because it states that LL acquired a one-
fifth interest in the firm directly from NN and VV and no goodwill is to be recorded. In other
words, this was a transaction between partners.

9. PP contributed P24,000 and CC contributed P48,000 to form a partnership, and they agreed to
share profits in the ratio of their original capital contributions. During the first year of operations,
they made a profit of P16,290; PP withdrew P5,050 and CC P8,000. At the start of the following
year, they agreed to admit GG into the partnership. He was to receive a one-fourth interest in
the capital and profits upon payment of P30,000 to PP and CC, whose capital accounts were to
be reduced by transfers to GG’s capital account of amounts sufficient to bring them back to their
original capital ratio.

How should the P30,000 paid by GG be divided between PP and CC?

PP CC TOTAL

Capital balances before net income P24,000 P48,000 P72,000


Net income (24:48) or (1/3:2/3) 5,430 10,860 16,290
Drawings ( 5,050) ( 8,000) ( 13,050)
Capital balances before admission P24,380 P50,860 P75,240

Amount paid P30,000


Less: Book value of interest acquired (P75,240 x ¼) 18,810
Gain of PP and CC P11,190

Therefore, the P30,000 cash should be allocated as follows:

Capital balances before admission P24,380 P50,860 P75,240


Required capital balances
[P & L ratio – 1/3:2/3 of P56,430
(P75,240 – P18,810)] 18,810 37,620 56,430
Transfer of capital to bring back
original ratio P 5,570 P13,240 P18,810
Add: Personal gain (refer above), 1/3:2/3 3,730 7,460 11,190
Personal cash distribution P 9,300 P20,700 P30,000
10. In the AA-BB partnership, AA and BB had a capital ratio of 3:1 and profit and loss ratio of 2:1,
respectively. The bonus method was used to record CC’s admittance as a new partner. What
ratio would be used to allocate, to AA and BB, the excess of CC’s contribution over the
amount credited to CC’s capital account? AA and BB’s old profit and loss ratio

The bonus method implied that the old partner either receive a bonus from the new partner, or
they pad a bonus to the new partner. In this case, CC, the new partner invested an amount in
excess of the amount credited to CC’s capital account. Accordingly, the excess should be
treated as a bonus to AA and BB. This bonus should be treated as an adjustment to the old
partners’ capital accounts and should be allocated by using AA and BB’s old profit and loss
ratio.

11. Merlin, a partner in the Comelot Partnership, has a 30% participation in partnership profits and
losses. Merlin’s capital account has a net decrease of P1,200,000 during the calendar year
2013. During 2013, Merlin withdrew P2,600,000 (charged against his capital account) and
contributed property valued at P500,000 to the partnership.

What was the net income of the Camelot Pertnership for the year 2013? P3,000,000

Withdrawals P(2,600,000)
Investment 500,000
Share in net income (balancing figure) 900,000
Net (decrease) increase P(1,200,000)

Net income of the partnership: P900,000 / 30% P 3,000,000

12. X, Y and Z, a partnership formed on January 1, 2013 had the following initial investment:

X - P100,000
Y - 150,000
Z - 225,000

The partnership agreement states that profits and losses are to be shared equally by the
partners after consideration is made for the following:

 Salaries allowed to partners: P60,000 for X, P48,000 for Y and P36,000 for Z.
 Average partner’s capital balances during the year shall be allowed 10%.

Additional information:

 On June 30, 2013 X invested an additional P60,000


 Z withdrew P70,000 from the partnership on September 30, 2013.
 Share in the remaining partnership profit was P5,000 for each partner.

The total partnership capital on December 31, 2013 was: P672,750

Capital, January 2013 P475,000


Add: Investment 60,000
Net income 207,750
Total P742,750
Less: Withdrawals 70,000
Capital, December 31, 2013 P672,750

13. Jones and smith formed a partnership with each partner contributing the following items:

Jones Smith
Cash P 80,000 P 40,000
Building – cost to Jones 300,000
- Fair value 400,000
Inventory – cost to Smith 200,000
- Fair value 280,000
Mortgage payable 120,000
Accounts payable 60,000

Assume that for tax purposes Jones and Smith agree to share equally in the liabilities assumed
by the Jones and Smith partnership.

What is the balance in each partner’s capital account for financial accounting purposes?

Jones Smith
Assets at fair value
Jones: P800,000 + P400,000 P480,000
Smith: P400,000 + P280,000 P320,000
Less: Liabilities assumed 120,000 60,000
Capital P360,000 P260,000

14. On January 2, 2013, BB and PP formed a partnership. BB contributed capital of P175,000.00


and PP, P25,000.00. They agreed to share profits and losses 80% and 20% respectively. PP is
the general manager and works in the partnership full time and is given a salary of P5,000.00 a
month; an interest of 5% of the beginning capital (of both partner) and a bonus of 15% of net
income before the salary, interest and the bonus.

The profit and loss statement of the partnership for the year ended December 31, 2013 is as
follows:

Net Sales P 875,000


Cost of goods sold 700,000
Gross Profit P 175,000
Expenses (including the salary, interest and the bonus) 143,000
Net income P 32,000

The amount of bonus to PP in 2013 amounted to:

Bonus = .15 (NI before salaries, interest and bonus)


B = ,15 (NI after salaries, interest and bonus + salaries + interest + bonus)
B = .15 [P32,000 + (5,000 x 12) + (5% x P200,000) + B]
B = .15 [P32,000 + P60,000 + P10,000 + B]
B = .15 [P102,000 + B]
B = P15,300 + .15B
B - .15 B = P15,300
.85 B = P15,300
B = P15,000/8.5
B = P18,000

15. A, B, and C are partners in an accounting firm. Their capital account balances at year-end were
A P90,000; B P110,000 and C P50,000. They share profits and losses on a 4:4:2 ratio, after the
following special terms:

 Partner C is to receive a bonus of 10% of net income after the bonus.


 Interest of 10% shall be paid on that portion of a partner’s capital in excess of
P100,000.
 Salaries of P10,000 and P12,000 shall be paid to partners A & C respectively.

Assuming a net income of P44,000 for the year, the total profit share of Partner C was:

Bonus* P 4,000 P 4,000


Interest: 10% (P110,000 – P100,000) P1,000 - 1,000
Salaries P10,000 - 12,000 22,000
Balance: 4:4:2 3,400 17,000
P19,400 P44,000

*Bonus = 10% (NI – Bonus)


B = .10 (P44,000 – B)
B = P4,400 - .10B
1.10B = P4,400
B = P4,000

16. CC and DD are partners who share profits and losses in the ratio of 7:3, respectively. On
October 21, 2013, their respective capital account is as follows:

CC P35,000
DD 30,000
P65,000

On that date they agreed to admit EE as a partner with a one-third interest in the capital and
profits and losses, and upon his investment of P25,000. The new partnership will begin with a
total capital of P90,000. Immediately after EE’s admission, what are the capital balance of
CC, DD and EE, respectively?

Total agreed capital of the new partnership P90,000


Less: Total contributed capital (P35,000 + P30,000 + P25,000) 90,000
P 0

Following the admission of EE, the partnership began with a total capital of P90,000, and EE
received a one-third interest; therefore his capital balance must be credited for P30,000
(P90,000 x 1/3). But EE contributes only P25,000, so the P5,000 difference represents bonus
(P30,000 – P25,000) must be debited and allocated to the old partners in the ratio of 7:3:

CC [P35,000 – (70% x P5,000)] P31,500


DD [P30,000 – (30% x P5,000)] 28,500
EE (P90,000 x 1/3) 30,000
Total agreed capital P90,000

17. The partners’ capital (income-sharing ratio in parentheses) of Nunn, Owen, Park and Quan LLP
on May 31, 2014 were as follows:

Nunn (20%) P 60,000


Owen (20%) 80,000
Park (20%) 70,000
Quan (40%) 40,000
Total partner’s capital (20%) P250,000

On may 31, 2014, with the consent of Nunn, Owen and Quan:
 Sam Park retired from the partnership and was paid P50,000 cash in full
settlement of his interest in the partnership.
 Lois Reed was admitted to the partnership with a P20,000 cash investment for a
10% interest in the net assets of Nunn, Owen and Quan.

The capital account to be credited to Reed is:

Total capital before retirement P250,000


Less: Retirement of Park 70,000
Add: Bonus to remaining partners due to retirement of Park 20,000
Capital balance before the admission of Reed P200,000
Add: Cash investment of Reed 20,000
Total agreed capital of the partnership (equal to the contributed capital) P220,000
Multiplied by: Interest acquired 10%
Capital account to be credited to Reed P 22,000

18. MM, NN and OO are partners with capital balances on December 31, 2013 of P300,000 and
P200,000, respectively. Profits are shared equally. OO wishes to withdraw and it is agreed that
OO is to take certain equipment with second-hand value of P50,000 and a note for the balance
of OO’s interest. The equipment are carried on the books at P65,000. Brand new equipment
may cost P80,000.

Compute for: (1) OO’s acquisition of the second-hand equipment will result to reduction
in capital; (2) the value of the note that will OO get from the partnership’s liquidation.

(1) Reduction in capital


Equipment at carrying value P 65,000
Equipment at second-hand value (fair value) 50,000
Decrease in equipment P 15,000
Multiply by: Profit and Loss ratio of MM, NN and OO 1/3
Reduction in capital P 5,000

(2) Notes payable to OO:


Unadjusted capital of OO P200,000
Less: Share in the decrease of equipment 5,000
Adjusted capital of OO P195,000
Less: Equipment received at second-hand value 50,000
Value of notes payable P145,000

19. JJ & KK partnership’s balance sheet at December 31, 2012, reported the following:

Total assets P100,000


Total liabilities 20,000
JJ, capital 40,000
KK, capital 40,000

On January 2, 2013, JJ and KK dissolved their partnership and transferred all assets and
liabilities to a newly-formed corporation. At the date of incorporation, the fair value of the net
assets was P12,000 more than the carrying amount on the partnership’s books, of which P7,000
was assigned to tangible assets and P5,000 was assigned to goodwill. JJ and KK were each
issued 5,000 shares of the corporation’s P1 par value ordinary share. Immediately following
incorporation, share premium/additional paid-in-capital in excess of par should be
credited for:

Carrying value of net assets (P100,000 – P20,000) P80,000


Add: Adjustments to reflect fair value 12,000
Fair value P92,000
Less: Common stock, P1 par (5,000 shares x 2 x P1) 10,000
Additional paid-in capital/share premium P82,000

20. After operating for five years, the books of the partnership of Bo and By showed the following
balances:

Net assets P169,000


Bo, capital 110,500
By, capital 58,500
If liquidation takes place at this point and the net assets are realized at book value, the
partners are entitled to: Bo to receive P110,500 & By to receive P58,500

The non-cash assets are realized at book value therefore; there is no gain or loss, in which case
partners are entitled to received an amount equivalent to their capital interest.

INSTALLMENT SALES

 On January 1, 2011, Art company sold its idle plant facility to Tony, Inc. for
P1,050,000.On this date, the plant had a depreciated cost of P735,000. Tony paid P150,000
cash on January 1, 2011 and signed a P900,000 note bearing interest at 10%. The note was
payable in three annual installments of P300,000 beginning January 1, 2012. Art appropriately
accounted for the sale under the installment method. Tony made a timely payment of the first
installment on January 1, 2012 of P390,000 which included interest of P90,000 to date of
payment.

At December 31, 2012, Art hast deferred gross profit of: P180,000

Installment sales – 2011 P1,050,000


Less: Downpayment – 1/1/2011 150,000
Installment Collections – 1/1/2011 (P390,000 – P90,000) 300,000
Installment Accounts Receivable, 12/31/2012 P 600,000
Multiplied by: Gross profit rate
(P1,050,000 – P735,000) /P1,050,000 30%
Deferred gross profit end of 2012 P 180,000

2. On October 1, 2011, Rodel Corporation, a real estate developer, sold land to Gerry Company
for P5,000,000. Gerry paid cash of P600,000 and signed a ten year P4,400,00 note bearing
interest at 12%. The carrying amount of the land was P4,000,000 on the date of sale. The note
was payable in forty quarterly principal installments of P110,000 beginning January 2, 2012.
Rodel appropriately accounts for the sale under the cost recovery method. On January 2, 2012,
Gerry paid the first principal installment of P110,000 and interest of P132,000.

For the year ended December 31, 2012, what total amount of income should Rodel
recognize from the land sale and the financing: P 0

Carry the value of land, 10/1/2011 P4,000,000

Less: Collections from October 1, 2011 to October 1, 2012


(Principal and interest) 1,548,200
Unrecovered cost 12/31/2012 P2,451,800

Profit from sale and interest income recognized P 0

3. Asser Computer Co. began operation at the beginning of 2012. During the year, it had cash
sales of P6,875,000 sales on installment basis of P16,500,000. Asser adds a markup on cost of
25% on cash sales and 50% on installment sales. Installment receivable at the end of 2012 is
P6,600,000.

Total realized gross profit for 2012 is: P4,657,000

Realized Gross Profit:


Cash sales: P6,875,000 x 25/125 P1,375,000
Installment sales:
Installment Sales – 2012 P16,500,000
Less: Installment Accounts
Receivable, end of 2011 6,600,000
Collections on installment sales 9,900,000
Multiply by: gross profit – 2012 50/150 3,300,000
Total realized gross profit – 2012 P4,675,000

4. The Molino Furniture Company appropriately used the installment sales method in accounting
for the following installment sale. During 2011, Molin o sold furniture to an individual for P3,000
at a gross profit of P1,200. On June 1, 2011, this installment account receivable had a balance
of P2,200 and it was determined that no further collections would be made . Molino, therefore,
repossessed the merchandise. When reacquired, the merchandise was appraised as being
worth only P1,000. In order to improve its salability, Bengal incurred costs of P100 for
reconditioning. Normal profit on resale is P200.

What should be the loss on repossession attributable to this merchandise: P320

Appraised value before reconditioning cost P1,000


Less: Unrecovered cost:
Installment Accounts Receivable, unpaid balance P2,200
Less deferred gross profit [P2,200 x (1,200/3,000)] 880 1,320
Loss on repossession P 320
5. Spicer Corporation has a normal gross profit on installment sales of 30%. A 2009 sale resulted
in a default early in 2011. At the date of default, the balance of the installment receivable was
P24,000, and the repossessed merchandise had a fair value of P13,500.

Assuming the repossessed merchandise is to be recorded at fair value, the gain or loss
on repossession should be: P3,300 loss

Fair value before reconditioning cost P13,500


Less: Unrecovered cost –
Installment accounts receivable,
Unpaid balance P24,000
Less: Deferred gross profit (30% x P24,000) 7,200 16,800
Loss on repossession P(3,300)

6. Fryman Furniture uses the installment-sales method. No further collections could be made on
an account with a balance of P18,000. It was estimated that the repossessed furniture could be
sold as is for P5,400, or for P6,300 if P300 were spent reconditioning it. The gross profit rate on
the original sale was 40%.

The loss on repossession was: P4,800

Fair value before reconditioning costs (P6,300 – P300) P6,000*


Less: Unrecovered cost –
Installment accounts receivable,
Unpaid balance P8,400
Less: Deferred gross profit (70% x P18,000) 5,880 2,520
Gain on repossession P180

21.

(1) (2) (3) (4)


1% x (4) (1) – (2) (4) – (3)
Dates Collection Interest Principal Unpaid Balance
9/30/11 P3,600,000
9/30/11 P1,600,000 P-0- P1,600,000 2,000,000
10/31/11 220,000 20,000 *200,000 1,800,000
11/30/11 218,000 18,000 200,000 1,600,000
12/31/11 216,000 16,000 200,000 1,400,000
1/31/12 214,000 14,000 200,000 1,200,000
*monthly collections as to principal.
7. On January 1, 2011, Janette Company sold 20,000 square meters of farmland for P600,000 to
Michelle, taking in exchange a 10% interest bearing note Janette Company purchased the
farmland in 2011 at a cost of P500,000. The note will be paid in three installments of P241,269
including interest each on December 31,2011, 2012, and 201. Shortly, after the sale Janette
Company learns distressing news about Michelle’s financial circumstances and because
collection is so uncertain and decides to account for the sale using the cost recovery method.

Determine the Realized Gross Profit and Interest Income for the year 2012, and
Unrecovered Cost as of December 31, 2012, respectively: P -0-; P -0-; P 17,462

Cost, January 1, 2011 ……………………………………………… P500,000


Less: Collections including interest – 2011 ……………………… 241,269
Collections including interest -2012 ……………………… 241,269

Unrecovered Cost, December 31, 2012 ……………………… P 17,462

Under the cost recovery method, no income is recognized on a sale until the
cost of the item sold is recovered through cash receipts. All cash receipts, both
interest and principal portions, are applied first to the cost of the items sold.
Then, all subsequent receipts are reported as revenue. Because all costs have
been recovered, the recognized revenue after the cost recovery represents
income (interest and realized gross profit). This method is used only when the
circumstances surrounding a sale are so uncertain that earlier recognition is
impossible.

8. Johnson Enterprises uses the cost recovery method for all installment sales. Complete the
following table:

2010 2011 2012


Installment sales P80,000 P95,000 P?
Cost of installments sales ? 56,050 68,250
Gross profit percentage 38% ? 35%
Cash collections:
2010 sales 25,000 46,000 5,600
2011 sales 22, 800 ?
2012 sales 32,550
Realized Gross Profit on
Installment Sales ? ? 16, 050

The Installments sales in 2012: 105,000.00

Installment sales:
Cost of Installments Sales: P68,250 / (100%-35%)…… P105,000

9. Using the same information in No. 9, the collection in 2012 for 2011 sales : P43, 700
Cost of installments sales for 2011 Installments Sales ………… P56, 050
Less: Collections in 2011 for 2011 Installment Sales …………… 22, 800
Collections in 2012 for 2011 Installment Sales
( balancing figure) ………………………………… 43, 700
RGP on Installments Sales in 2012 for 2011
Installment Sales ………………………… ………………… P10,450*

*RGP on I/S in 2012 …………………………………………………… P16, 050


Less: RGP on I/S in 2012 for 2010 I/S since Cost of P49,600 (No. 2)
Is already recovered in 2011 (equivalent to collection)………. 5,600
RGP on I/S in 2012 for 2011 Installment Sales ……………………. P10, 450

10. Using the same information in No. 9, the realized gross profit on installment sales in
2011: 22, 400

Cost of installments sales for 2010 Installment Sales ………… P49,600


Less: Collections in 2010 for 2010 Installment Sales …………. 25,600
Collections in 2011 for 2010 Installment Sales ………… 46,400
RGP on I/S in 2011 for 2010 Installments Sales …………………P22,400
RGP on I/S in 2011 for 2011 I/S:
Cost of installment sales for 2011 Installment
Sales ………………………………………P56, 050
Less: Collections in 2011 for 2011 I/S ………… 22, 800
Unrecovered Cost in 2011 for 2011 I/S … P33,250 -0-
RGP on I/S in 2011 ………………………………………………… P22,400

11. Using the same information in No. 9, the realized gross profit on installment sales in
2010: Zero

Cost is not yet fully recovered, therefore, no profit should be recognized


Cost of installment sales for 2010 Installment Sales ………... P49,600
Less: Collections in 2011 for 2010 Installment Sales ……… 25,600
Unrecovered cost in 2010 for 2011 Installment Sales …… P24,000

12. The Precious Appliance Company started business on January 1, 2013. Separate accounts
were established for installment and cash sales.

On installment sales, the contract price is 106% of the cash sale price. A standard installment
contract is used whereby a down payment of ¼ of the installment price is required, with the
balance payable in 15 equal monthly installments. The interest charged per month is 1% of the
unpaid cash sales price equivalent. It is recognized in the period earned.

Installments receivable and installment sales are recorded at the contract price. When contracts
are defaulted, the unpaid balances are charged to Bad Debt Expense. Sales of defaulted
merchandise are credited to Bad Debt Expense.
The following data show the results of transactions in 2013:

Sales:
Cash sales P126,000
Installment sales 265,000
Repossessed sales 230
Merchandise inventory, January 1, 2013 58,060
Purchases 209,300
Merchandise inventory, December 31, 2013:
New merchandise 33,300
Repossessed inventory 180
Cash collections on installment contracts:
Down payments 66,250
Subsequent installment including interest of P9,252.84
(average of six monthly installment on all contracts,
except on defaulted contracts) 79,341

Five contracts totaling to P1060 were defaulted after 3 monthly installment payments.

The gross profit percentage in 2013 based on cash sales price equivalent is: 37.75%

Sales P 126,000
Installment Sales at Cash Sales (P265,000 / 1.06) 250,000
Total Sales at Cash Sales Price P 376,000
Less: Cost of Goods Sold:
Merchandise Inventory, 1/1/13 P 58,060
Add: Purchases 209,300
Cost of goods available for sale P 267,360
Less: Merchandise Inventory, 12/31/13 33,300 234,060
Gross Profit P 141,940

Gross Profit Percentage: P141,940 / P376,000 37.75%

13. The following selected accounts are taken from the trial balance on December 31, 2012 of
Tacloban Company:

Accounts receivable – charge sales P 75,000


Installment receivables – 2010 15,000
Installment receivables– 2011 45,000
Installment receivables – 2012 270,000
Merchandise inventory 52,500
Purchases 390,000
Freight-in 3,000
Repossessed merchandise 15,000
Repossessed loss 24,000
Cash sales P 90,000
Charge sales 180,000
Installment sales 446,400
Deferred gross profit – 2010 22,200
Deferred gross profit – 2011 39,360

Additional information:

a. Gross profit rate on 2010 installment sales was 30% and for 2011, the rate was 32%.
b. Installment sales prices exceed cash sales prices by 24% while charge sales prices
exceed cash sales prices by 20%.
c. The entry for repossessed goods was:
Repossessed merchandise P15,000
Repossession loss 24,000
Installment receivables – 2010 P18,000
Installment receivables – 2011 21,000
d. Merchandise on hand at the end of 2012 (new and repossessed was P70,500.

(1) If all sales were on cash basis, the total sales for 2012, and (2) The cost of goods sold on
installment sales for 2012: (1) 600,000; (2) 234,000

(1) Total Sales for 2012:


Cash sales (equivalent to 100%) P 90,000
Installment sales (P446,400/124% = equivalent to 100%) 360,000
Charges Sales (P180,000/120% = equivalent to 100%) 150,000
P600,000

(2) Cost of Installment Sales for 2012:


Merchandise Inventory, 12/31/2011 P 52,500
Add: Purchases 390,000
Freight-in 3,000
Repossessed Merchandise 15,000
Cost of goods available for sale P460,500
Less: Merchandise Inventory, 12/31/2012
(new repossessed) 70,500
Cost of goods sold P390,000
Multiplied by: Based on sales amount (equivalent to
cash sales price which is 100% 360/600
Cost of installment sales P234,000

14. Using the same information in No. 14, The cash collections on installment Sales for –

2010 sales 2011 sales 2012 sales

Installment Accounts Receivable 1/1


2012 (installment Sales) P446,400
2011 : P39,360 / 32% P123,000
2010 : P22,200 / 30% P 74,000
Less: Installment Accounts
Receivable, 12/31/2012 15,000 45,000 270,000

Decrease in installment Accounts


Receivable P 59,000 P 78,000 P176,400
Less: Defaults, unpaid balance 18,000 21,000 ________
Collections in 2012 P41,000 P 57,000 P176,400

15. The trial balance of LOL Appliance Corporation as of the end of the fiscal year on September
30, 2012 is:

Debit
Accounts receivable P 100,000
Accounts payable P 100,000
Allowance for depreciation 33,750
Capital stock 125,000
Cash 46,250
Deferred gross profit – 2011 50,000
Equipment 112,500
Installment contract receivable – 2011 12,500
Installment contract receivable – 2012 150,000
Installment sales 375,000
Inventory, September 31, 2011 62,500
Loss on repossessions 3,750
Prepaid expenses 3,750
Purchases 435,000
Repossessions 2,000
Retained earnings 30,000
Sales __________ 312,500
Selling and administrative expenses 97,500
Total P1,026,250 P1,026,250

The post-closing trial balance on September 30, 2011 shows the following balances of certain
accounts:

Installment contract receivable – 2011 P 100,000


Deferred gross profit – 2011 50,000

The gross profit percentage on regular sales during the year was 30%.

The accountant made the following entry for a repossession on a sale of 2011 towards the end
of fiscal year:

Repossessions P2,500
Loss in repossessions 3,750
Installment contract receivable – 2011 P6,250

The inventory of new repossessed merchandise on September 30, 2012 amounted to P75,000

The total realized gross profit for the fiscal year September 30, 2012: P235,625

Installment contract receivable,


9/30/2011 (installment sales) P100,000 P375,000
Less: installment contract receivable,
9/30/2012 12,500 150,000
Decrease in installment contract
Receivable P 87,500 P225,000
Less: Defaults, unpaid balance 6,250 ________
Collections in 2012 P 81,250 P225,000
Multiplied by: Gross profit rate 50% 45%
Realized gross profit on installment sales P 40,625 P101,250 P141,875
Add: Gross profit on Cash Sales – 2012
P312,500 x 30% 93,750
Realized gross profit in 2012 P235,625

16. Using the same information in No. 16, the correcting entry for repossession made on a sale
of 2011 is:

Value of Repossessed Merchandise P2,500


Less: Uncovered Cost:
Installment Contract Receivable – 2011,
unpaid balance P6,250
Less: Deferred gross profit – 2011
(P6,250 x 50%) 3,125 3,125
Loss on repossession P( 625)

Correcting Entry:
Deferred gross profit – 2011 3,125
Loss on repossession (P3,750 – P625) 3,125

Entry Made:
Repossessions 2,500
Loss on Repossessions 3,750
Installment contract receivable – 2011 6,250

Correct Entry:
Repossessions 2,500
Deferred gross profit – 2011 3,125
Loss on repossessions 625
Installment contract receivable – 2011 6,250

17. Using the same information in No. 16, compute the net income for the fiscal year September
30, 2012: P137,500

Realized gross profit in 2012 P235,625


Less: Loss in repossession 625
Seling and administrative expenses 97,500
Net income P137,500

18. Sharon Company uses the installment sales method in accounting for its installment sales. On
January 1, 2013, Sharon Company had an installment accounts receivable from Rowena with a
balance of P18,000. During 2013, P4,000 was collected from Rowena. When no further
collection could be made, the merchandise sold to Rowena was repossessed. The merchandise
had a fair market value of P6,500 after the company spent for P600 for reconditioning of the
merchandise. The merchandise was originally sold with a gross profit rate of 40%.

Determine the gain or loss on repossession and cost of repossessed merchandise,


respectively: P2,500 loss; P6,500

Fair Market Value after reconditioning costs P6,500


Less: Reconditioning costs 600
Fair Market Value before reconditioning costs P5,900
Less: Unrecovered cost:
Unpaid balance (P18,000 – P4,000) P14,000
Less: Deferred gross profit (40% x P14,000) 5,600 8,400
Loss on realization P2,500

The cost of repossessed merchandise should amount to P6,500 (P5,900 + P600)


it should be noted that reconditioning costs is inventoriable.

19. On January 1, 2013, Janette Company sold 20,000 square meters of farmland for P600,000 to
Michelle, taking in exchange a 10% interest bearing note Janette Company purchased the
farmland in 2013 at a cost of P500,000. The note will be paid in three installments of P241,269
including interest each on December 31, 2013, 2014 and 2015. Shortly, after the sale Janette
Company learns distressing news about Michelle’s financial circumstances and because
collection is so uncertain and decides to account for the sale using the cost recovery method.

Determine the Realized Gross Profit and Interest Income for the year 2014, and
Uncovered Cost as of December 31, 2014, respectively.

Cost, January 1, 2013 P500,000


Less: Collections including interest – 2013 241,269
Collections including interest – 2014 241,269
Uncovered Cost, December 31, 2014 P 17,462

Activity Based Costing (ABC) and Just in Time Costing System (JIT)

1. Uratex Company manufactures a variety of classroom chairs. Its job-costing system uses an
activity-based approach. There are two direct-cost categories (direct materials and direct labor)
and three indirect cost pools. The cost pools represent three activity areas at the plant.

Manufacturing Budgeted Cost Cost Driver Used Cost-allocation


Activity Area for 2001 as Allocation Base Rate

Materials Handling P 200,000 Parts P 0.25


Cutting 2,000,000 Parts 2.50
Assembly 2,000,000 Direct labor hours 25.00
Two styles of chairs were produced in March, the high school chair, and the college chair. Their
quantities, direct material costs, and other data for March 2014 are as follows:

Units Direct Material Number of Direct Manufacturing


Produced Costs Parts Labor Hours

High School Chair 5,000 P 600,000 100,000 7,500


College Chair 100 25,000 3,500 500

The direct labor rate is P20 per hour. Assume no beginning or ending inventory.

What are the unit cost of the high school chair and the college chair?

P242.50 and P570.25 respectively

High school chair :

Direct Materials P 600,000


Direct labor (7,500 x P20) 150,000
Overhead:
Material handling (100,000 x P0.25) P 25,000
Cutting (100,000 x P2.50) 250,000
Assembly (7,500 x P25) 187,500 462,500
Total cost P 1,212,500
Divided by: Units produced 5,000
Unit Cost P 242.50

College chair:

Direct Materials P 25,000


Direct Labor (500 x P20) 10,000
Overhead:
Material handling (3,500 x P0.25) P 875
Cutting (3,500 x P2.50) 8,750
Assembly (500 x P25) 12,500 22,125
Total cost P 57,125
Divided by: Units produced 100
Unit costs P 571.25
2. The Manila Company manufactures and sells packaging machines. It recently used an activity-
based approach to refine the job costing system at its Bulacan plant. The resulting job costing
system has one direct-cost category (direct materials) and for indirect manufacturing cost pools.
These four indirect cost pools and their allocation bases are:

Indirect Manufacturing Cost-Allocation Budgeted Cost-


Cost Pool Base Allocation Base

1. Material Handling Component parts P8 per part


2. Machining Machine-hours P68 per hour
3. Assembly Assembly-hours P75 per hour
4. Inspection Inspection-hours P104 per hour

Manila company recently sold 50 can packaging machines to Ilocos Company. Each machine
has direct material costs of P3,000 requires 50 component parts, 12 machine-hours, 15
assembly-hours, and 4 inspection hours.

Manila Company’s previous costing system had one direct-cost category (direct materials)and
one indirect-cost category (manufacturing overhead allocated at the rate of P100 per assembly-
hours).

In comparison to the traditional costing system used by Manila Company, the total
manufacturing cost of the machines sold under the ABC is: P114,850 higher

Traditional costing system:


Direct Materials (50 x P3,000) P 150,000
Overhead (50 x 15) x P100 75,000
Total cost P 225,000

Activity based costing system:


Direct Materials P 150,000
Overhead:
Material handling (50 x 50) x P8 20,000
Machining (50 x 12) x P68 40,800
Assembly (50 x 15) x P75 56,250
Inspection (50 x 14) x P104 72,800 P 339,850
Higher P 114,850

3. Tamiya Corporation has used a traditional costing system to apply quality control costs
uniformly to all products at a rate of 14.5% of direct labor cost. Monthly direct labor cost for its
Product X is P275,000. In an attempt to distribute quality control costs more equitable, Tamiya
is considering activity-based costing (ABC). The June data shown below have been gathered
for Product X.
Activity Cost Driver Cost Rates Quantity

5. Material handling Type of materials P115 per types 12 types


6. Inspection Number of units P1.40 per unit 17,500 units
7. Product certification Per order P770 per order 25 orders

What is the monthly quality control assigned to product X using the ABC?
P5,255 higher than the traditional costing system

ABC:
Material Handling (115 x 12) P 1,380
Inspection (P1.40 x 17,500) 24,500
Product certification (P770 x 25) 19,250 P 45,130
Traditional Costing (P275,000 x 14.5%) 39,875
ABC higher than the traditional costing by P 5,255

4. Malaysia Inc. accumulated the following cost information for its two products, A and B.

Product A Product B
Units Produced 2,000 1,000
Total direct labor hours 5,000 20,000
Set-up cost per batch P 1,000 P 2,000
Batch size 100 50
Total set-up cost incurred P 20,000 P 40,000
Direct labor hour per unit 2 1
A traditional costing system would allocate setup costs on the basis of direct labor hours. An
ABC system would trace costs by spreading the cost per batch over the units in a batch.

What is the setup cost per unit of Product A under each costing system.

Traditional Costing ABC


P4.80 P10

Traditional costing (60,000 ÷ 25,000 x 2 direct labor hours) P 4.80


ABC (1,000 ÷ 100) P 10.00

5. Believing that its traditional cost system may be providing misleading information. BMW
Company is considering an activity based costing approach. It now employs a full cost system
and has been applying its manufacturing overhead on the basis of machine hours.

The company plans on using 50,000 direct labor hours and 30,000 machine hours in the coming
year. The following data show the manufacturing overhead that is budgeted.

Activity Cost Driver Budgeted Activity Budgeted Cost


Material Handling No. of parts 6,000,000 P 720,000
handled
Set up costs No. of setups 750 315,000
Machining costs Machine hours 30,000 540,000
Quality control No. of batches 500 225,000

Cost, sales, and production data for one of the company’s product for the coming year are as
follows:

Prime Costs:
Direct material cost per unit P4.40
Direct labor cost per unit, .05 direct
hour@P15 per hour 0.75
Sales and production data:
Expected sales 20,000 units
Batch size 5,000 units
Setups 2 per batch
Total parts per finished unit 5 parts
Machine hours required 80 machine hours per batch

If the company employs an activity-based costing system, the cost per unit for the
product described for the coming year will be: P6.30

Overhead rates:
Material Handling (P720,000 ÷ 6,000,000 parts) P 0.12
Setup costs (P315,000 ÷ 750 setups) 420.00
Machining costs (P540,000 ÷ 30,000 hours) 18.00
Quality control (P225,000 ÷ 500 batches) 450.00
Overhead costs:
Material handling (20,000 units x 5 parts) x P0.12 P 12,000
Setup costs activity (20,000 units/ 5,000 x 2 setups) x P420 3,360
Machining activity (20,000 units/ 5,000 x 80 hrs.) x P18 5,760
Quality Control activity (20,000 units/ 5,000) P450 1,800
Total P22,920

Overhead cost per unit (P22,920/20,000 units) P 1.15


Direct material cost per unit 4.40
Direct labor cost per unit 0.75
Total unit cost P 6.30

6. Edsa Company has materials cost in the June 1 Raw and in Process account of P10,000.
Materials received during June of P205,000 and materials cost in June 30 Raw and in Process
account of P12,500.

The amount to be backflushed from Raw and In process account to Finished Goods
account at the end of June would be: P202,500

Raw and in Process inventory, June 1 P 10,000


Materials received during June 205,000
Raw and in Process inventory, June 30 (12,500)
Amount to be backflushed (raw materials used) P 202,500

7. The Love Company seeks to streamline the costing system at its Manila plant. It will use a
backflush costing system with three trigger points:

 Purchase of raw materials


 Completion of finished goods
 Sale of finished goods

There are no beginning inventories. The following data pertain to April 2014:

Raw materials purchased P880,000


Raw materials used 850,000
Conversion cost incurred 422,000
Conversion allocated to finished goods 400,000
Costs transferred to finished goods 1,250,000
Costs of goods sold 1,190,000

Assume no material variances. The balance of RIP account at the end of April 2014 is:
P 30,000

Raw materials purchased (Debit to RIP) P880,000


Raw materials used (Credit to RIP) 850,000
Balance of RIP account P 30,000

8. The Maganda Manufacturing Company uses a Raw and in process (RIP) Inventory account. At
the end of each month, all inventories are counted, their conversion costs components are
estimated, and inventory account balances are adjusted accordingly. Raw materials cost is
backflushed from RIP account to Finished Goods account. The following data is for the month of
August:

Beginning balance of RIP account P 38,700


Conversion costs incurred 4,800
Raw materials purchased 680,000
Conversion costs allocated 5,300
Ending balance of RIP account 41,900

The amount of direct materials and conversion costs to be backflushed to finished goods
are: P676,800 and P5,300 respectively

RIP account, beginning balance P 38,700


Raw materials purchased 680,000
RIP account, ending balance (41,900)
Direct materials to be backflushed P676,800

Conversion costs allocated to be backflushed P 5,300

9. The Action Corporation manufactures electrical meters. For May, there were no beginning
inventories of raw materials and no beginning and ending work in process. Action uses a JIT
manufacturing system and backflush costing with three trigger points for making entries in the
accounting system:

 Purchase of raw materials – debited to Raw and in Process account


 Completion of finished goods – debited to Finished Goods account
 Sale of finished goods

Action’s May standard cost per meter are direct materials, P25; and conversion costs,
P20. The following data apply to May manufacturing:
Raw materials and components purchased P550,000
Conversion costs incurred P440,000
Number of finished units manufactured 21,000
Number of Finished units sold 20,000

The balances of Raw and in Process and Finished Goods inventory accounts at the end
of May are: P25,000 and P45,000 respectively

Raw materials purchased – Dr. to RIP P550,000


Direct materials to be backflushed – Cr. To RIP (21,000 x P25) 525,000
Balance of RIP account P 25,000
Cost of Completed units (21,000 units x P45) P945,000
Cost of Goods sold (20,000 units x P45) 900,000
Balance of Finished Goods inventory account P 45,000

10. The Malakas Company produces telephones. For June, there were no beginning inventory of
raw materials and no beginning and ending work in process. Malakas uses a JIT manufacturing
system and backflush costing with two trigger points for making entries in its accounting system:

 Purchase of raw materials


 Sales of finished goods

Malakas’ standard cost per unit of telephone in June are direct materials, P26; and conversion
costs, P15. The following data apply to June production:

Raw materials purchased P5,300,000


Conversion costs incurred 3,080,000
Number of finished units manufactured 200,000
Number of finished units sold 192,000

The balances of Raw and in Process and Cost of Goods Sold accounts at the end of June
are: P308,000 and P7,872,000 respectively

Raw materials purchased – Dr. to RIP P5,300,000


Direct materials to be backflushed (192,000 units x P26) –
Cr to RIP 4,992,000
Balance of RIP P 308,000

Cost of Goods Sold (192,000 units x P41) P7,872,000

11. Delta Machine Tool Incorporated produces varied product lines without the use of direct labor.
An extensive setup procedure is required. Because no single base for a predetermined
overhead rate will provide Delta with reliable product cost information, overhead is classified into
two cost pools and two predetermined overhead rates are used. For 2014, it is estimated that
total overhead costs will consist of P525,000 of overhead related to setups and P900,000 of
overhead related to machine usage. Total machine usage is expected to be 3,600 hours for the
year, and the total number of setups is expected to be 300.

Job RST required parts and materials costing P56,000, 70 hours of machine time, and four
setups.

What is the cost of Job RST? P80,500

Overhead Rates:
Per Machine Hour: P900,000/3,600 machine hours P 250
Per Setup: P525,000/300 setups P 1,750

Cost of Job RST:


Parts and materials P56,000
Applied overhead:
Machine hours (70 x P250) P17,500
Setups (4 x P1,750) 7,000 24,500
Total cost of Job RST P80,500

12. The Hudy Manufacturing Company uses a Raw and In Process (RIP) inventory account and
expensed all conversion costs to the cost of goods sold account. At the end of each month, all
inventories are counted, their conversion cost components are estimated, and inventory account
balances are adjusted accordingly. Raw materials cost is backflushed from RIP to finished
goods. The following information is for the month of April:

Beginning balance of RIP account, including


P1,400 conversion cost P 31,000
Raw materials received on credit 367,000
Ending RIP inventory per physical count, including
P1,800 conversion cost estimate 33,000

What is the amount of materials used to be backflushed from RIP to finished goods:
P365,400

Materials in beginning balance of RIP account


(P31,000 – P1,400) P 29,600
Add: Materials received on credit 367,000
Total P396,000
Less: materials in ending balance of RIP account
(P33,000 – P1,800) 31,200
Materials used to be backflushed from RIP to Finished Goods P365,400

13. The HPI manufacturing Company produces only for customer order and most work is shipped
within thirty-six hours after the receipt of an order. HPI uses a Raw and In Process (RIP)
inventory account and expensed all conversion cost to the cost of goods sold account. At the
end of each month, inventory is counted, its conversion cost component is estimated, and the
RIP account balance is adjusted accordingly. Raw material cost is backflushed from RIP to Cost
of Goods Sold. The following information is for the month of May:

Beginning balance of RIP account, including


P1,300 of conversion cost P 12,300
Raw materials received on credit 246,000
Ending RIP inventory per physical count, including
P2,100 conversion cost estimate 12,100

What is the amount of raw materials used to the backflushed from RIP to Cost of Goods
Sold? P247,000

Materials in beginning balance of RIP account


(P12,300 – P1,300) P 11,000
Add: Raw materials received on credit 246,000
Total 257,000
Less: materials in ending balance of RIP account
(12,100 – P2,100) 10,000
Materials used to be backflushed from RIP to Cost of Goods Sold P247,000

14. Mike Tuazon general manager of a highly automated coffee production plant in Bulacan has
provided the following information for transactions that accured during October. The production
plant uses the JIT costing system.

 Raw materials costing P300,000 were purchased.


 All materials costing P300,000 were requisitioned for production
 Direct labor costs of P200,000 were incurred
 Actual factory overhead costs amounted to P995,000
 Conversion costs allocated totaled P1,300,000. This includes the direct labor
cost.
 All units are completed and immediately sold

What is the over-allocated or under-allocated conversion costs for the month? P105,000

Actual factory overhead P 995,000


Direct labor costs incurred 200,000
Total actual conversion costs P1,195,000
Conversion costs allocated to production 1,300,000
Over-allocated conversion costs P 105,000

15. Using the same information, assuming no adjustment has been made for over-allocated or
under-allocated conversion cost, what is the balance of the cost of goods sold account on
October 31? P1,600,000

Materials used to be backflushed from RIP to CGS P 300,000


Applied conversion costs to production 1,300,000
Cost of Goods Sold balance, October 31 P1,600,000

16. Basilio Company has a cycle time of 3days, uses a RIP account, and changes all conversion
costs to Cost of Goods Sold. At the end of each month, all inventories are counted, their
conversion costs components are estimated, and inventory account balances are adjusted. Raw
material cost is bachflushed from RIP to Finished Goods. The following information is for June:

Beginning balance of RIP account, including


P3,000 of conversion costs P 29,250
Beginning balance of finished goods account including
P10,000 of conversion costs 30,000
Raw materials received on credit 562,500
Direct labor cost, P375,000; FOH applied P450,000 825,000
Ending RIP inventory per physical count, including
P4,500 of conversion costs 32,000
Ending finished goods inventory per physical count, including
P8,750 of conversion costs 26,250

What is the conversion costs of units sold in June? P824,750


Total conversion costs P 825,000
Adjustments: Increase in conversion cost in RIP
(P4,500 – P3,000) ( 1,500)
Decrease in conversion costs in FG
(P10,000 – P8,750) 1,250
Conversion costs of units sold in June P824,750

17. If Mike Company has material cost of P100,000 in the June 1 RIP inventory account, and
P12,500 in the June 30 RIP inventory account and the amount of raw materials used
backflushed from RIP inventory account on June 30 is P202,500.

What is the amount of raw materials purchased on credit for the month of June?
P205,000

Raw materials backflushed from RIP account P202,500


Materials in RIP inventory, June 30 12,500
Materials in RIP inventory, June 1 ( 10,000)
Raw materials purchased on credit during June P205,000

18. A company has identified the following overhead costs and cost drivers for the coming year:

Overhead Item Cost Drivers Budgeted Cost Bud. Act. Level

Machine setup # of setups P 20,000 200


Inspection # of inspections P 130,000 6,500
Material handling # of material moves P 80,000 8,000
Engineering Engineering hours P 50,000 1,000
P 280,000

The following information was collected on three jobs that were completed during the year:

Job 101 Job 102 Job 103


Direct materials P5,000 P12,000 P8,000
Direct labor P2,000 P 2,000 P4,000
Units completed 100 50 200
Number of setups 1 2 4
Number of inspections 20 10 30
Number of material moves 30 10 50
Engineering hours 10 50 10

Budgeted direct labor cost was P100,000 and budgeted direct material cost was P280,000

If the company uses activity-based costing, how much overhead cost should be
allocated to Job 101? P1,300

ABC allocates overhead costs more precisely than traditional methods. It identifies the
activities associated with the incurrence of costs, determines the cost driver for each activity,
and allocates cost accordingly. Thus the cost per setup is P100 (P20,000 / 200), per inspection
P20 (P130,000 / 6,500), per material move P10 (P80,000 / 8,000), and per engineering hour
P50 (P50,000 / 1,000). The overhead allocated to Job 101 is therefore P1,300 [(1 setup x P100
+ (20 inspections x P20) + (30 material moves x P10) + (10 engineering hours x P50)].

19. Using the same information in No. 18, compute the cost of each unit of Job 102 Activity-
Based Costing: P340

The overhead costs for the activities are P100 per setup, P20 per inspection, P10 per
material move, and P50 per engineering hour. Thus, overhead allocated to Job 102 is P3,000
[(2 setups x P100) + (10 inspections x P20) + (10 material moves x P10) + (50 engineering
hours x P50)]. The production cost of Job 102 is P17,000 (P12,000 DM + P2,000 DL + P3,000
OH), and the cost per unit is P340 (P17,000 / 50).

20. Using the same information in No. 19, assuming the company prices its products at 140%
of cost and the company uses Activity-Based Costing the price of each unit of Job 103
would be: P98

The costs per job for the activities are P100 per setup, P20 per inspection, P10 per
material move, and P50 per engineering hour. Overhead allocated to Job 103 is P2,000 [(4
setups x P100 + (30 inspections x P20) + (50 material moves x P10) + (10 engineering hours x
P50)]. Hence, the production cost of Job 103 is P14,000 (P8,000 DM + P4,000 DL + P2,000
OH), the cost per unit is P70 (P14,000 / 200), and the price is P98 (140% x P70).

FRANCHISE ACCOUNTING

1. On April 1, 2014, Motorola, Inc. entered into a franchise agreement with a local businessman.
The franchisee paid P45,000 and gave a P30,000, 8%, 3 year nots payable with interest due
annually on March 31. Motorola recorded the P75,000 initial franchise fee as revenue on April 1,
2014. On December 30, 2014, the franchisee decided not to open the outlet under Motorola’s
name. Motorola cancelled the franchisee’s note and refunded P24,000 less accrued interest on
the note, of the P45,000 paid on April 1, what entry should Motorola make on December 30,
2014?
Revenue from Franchise Fees 75,000
Interest Income 1,800
Cash 22,200
Notes Receivable 30,000
Revenue from Repossessed Franchise 21,000

Since initial franchise fee of P75,000 was initially recognized as revenue on April 1, then
to cancel the franchise, simply debit the Revenue account.
Interest Income: P30,000 x 8% x 9/12 = P1,800
Cash paid: P24,000 – P1,800 = P22,200
2. On January 1, 2014, Brownie Delight, Inc. entered into a franchise agreement with a company
allowing the company to do business under Brownie Delight’s name. Brownie Delights had
performed substantially all required services by January 1, 2014, and the franchisee paid the
initial franchise fee of P70,000 in full on that date. The franchise agreement specifies that the
franchisee must pay a continuing franchise fee of P6,000 annualy, of wich 20% must be spent
on advertising by Brownie Delight.

What entry should Brownie Delight make on January 1, 2014 to record the receipt of the
initial franchise fee and the continuing franchise fee for 2014?
Cash 76,000
Franchise Fee Revenue 70,000
Revenue from Continuing Franchise Fee 4,800
Unearned Franchise Fee 1,200

Initial Franchise Fee of P70,000:


CASH
Services YES
Period of Refund YES
P70,000
Status REVENUE

3. On September 1, 2014, Cindy Company entered into franchise agreement with two franchisees.
The agreements required an initial fee payment of P700,000 plus four P300,000 payments due
every four (4) months, the first payment due December 31, 2014. The market interest rate is
12%. The initial deposit is refundable until substantial performance has been completed. The
following table describes each agreement:

Services
Performed Total Costs
Probability of by Franchisor Incurred to
Franchisee Full Collection Dec. 31, 2014 Dec. 31, 2014
A Likely Substantially P700,000
B Doubtful 25% N/A

The present and future value tables for four (4) periods were as follows:

Present value of P1 0.8548


Present value of an ordinary annuity of P1 3.6299
Future value of P1 1.1699
Future value of an ordinary annuity of P1 4.2465

What amount of net income to be reported in 2014, assuming P1,000,000 was received
from each franchisee during the year: Franchisee A – P1,132,529; Franchisee B – P43,559

Franchise A:

Since there is already a substantial performance of the services it also follows


that the period of refund has been expired and the note is likely (reasonably assured) to be
collected, therefore, the full initial franchise fee is recognized as revenue.
The above circumstances imply that the full accrual method could be used:

Franchise Revenue:
Downpayment P 700,000
PV of installment: P300,000 x 3.6299 1,088,970
Total P1,788,970
Less: Cost of Franchise 700,000
Gross profit P1, 088,970
Add: Interest Income (P1,088,970 x 4%) 43,559
Net Income P1,132,529

Franchise B:

Since there is no substantial performance of services rendered it also follows that


the period of refund has not yet been expired and at the same time there is doubtful of
collection. Therefore, no initial franchise fee is recognized as revenue in 2014. However,
because the forst payment of P300,000 was made, interest income of P43,599 would be
recognized (P1,088,970 x 4%)

4. Saisaki Co. grants a franchise to Mity for an initial franchise fee of P1,000,000. The agreement
provides that Saisaki has the option within one year to acquire franchisee’s business, and it
seems certain that Saisaki will exercise this option.

On Saisaki’s books, how should the initial fee be recorded? Deferred and treated as
reduction inSaisaki investment when the option is exercised.

5. SSR Restaurant Inc., sold a fastfood restaurant franchise to Shar. The sale agreement, signed
on January 2, 2014, called for a P30,000 down payment plus two P10,000 annual payments,
representing the value of initial purchase services rendered by SSR Restaurant. In addition, the
agreement required the franchisee to pay 5% of its gross revenue to the franchisor; this was
deemed sufficient to cover the cost and provide a reasonable profit margin on counting
franchise services to be performed by SSR Restaurant. The restaurant opened early in 2014,
and its sales for the year amounted to P500,000. Assuming a 10% interest rate is
appropriate, SSR Restaurant’s 2014 total revenue will be: (the present value of an annuity
of P1 at 10% for two periods in 1.7355) P74,090

Franchise Revenue:

Initial franchise fee:


Downpayment P30,000
PV of Installment (P10,000 x 1.7355) 17,355 P47,355

Continuing Franchise Fee:


5% x P500,000 25,000
Total Franchise Revenue P72,355
Add: Interest Income (10% x P17,355) 1,735
Total Revenue P74,090
6. DJ Builders’ Enterprises, a franchisor, charges franchisees a “franchise fee: of P500,000. Of this
amount, a nonrefundable P200,000 is paid upon the signing of the contract with the balance
payable in three equal installments after each year thereafter. DJ Builders’ will assist in locating
a suitable business site, conduct a market study, oversee the construction of facilities, and
provide initial training for employees.

On December 1, 2013, DJ Builders’ signed a franchising agreement for the U-belt are. By the
end of 2013, it was determined that the substantial performance of the initial services had cost
DJ Builders’ a total of P150,000 and that collection of the balance of the franchise fee has been
reasonably assured.

In its 2013 income statement, DJ Builders’ should repot franchise revenue and net
income of P500,000 and P350,000 respectively.

Franchise Revenue P 500,000


Less: Cost of Franchise 150,000
Net income P 350,000

7. At the beginning of the year, AJD got the franchise of Tony’s, a known steak house of upscale
patronage. The franchise agreement required a P500,000 franchise fee payable P100,000 upon
signing of the franchise and the balance in four annual installments starting the end of the
current year. At present value using 12% as discount rate, the four installments would
approximate P199,650. The fees once paid are refundable. The franchise may be cancelled
subject to the provisions of the agreement. Should there be unpaid franchise fee attributed to
the balance of main fee, same would become due and demandable upon cancellation. Further,
the franchisor is entitled to a 5% fee on gross sales payable monthly within the first ten days of
the following month.

The Credit Investigation Bureau rated AJD as AAA credit rating. The balance of the franchise
fee was guaranteed by a commercial bank.

The first year of operations yielded gross sales of P9,000,000. Tony’s earned franchise
fees for the first year is: P450,000

Since the initial franchise fee is refundable, therefore, no amount from the P500,000
initial franchise fee be considered as revenue. The only revenue to be recognized is continuing
franchise fee of P450,000 (5% x P9,000,000).

8. Ruby Fruits Corporation enters into a franchise agreement with Rodel on June 1, 2011. As per
agreement, Rodel is to pay Ruby an up-front franchise fee of P1,000,000 and subsequent
annual franchise fees of P50,000 over the next four years. Cost of initial franchise services
rendered by Ruby during the year is P250,000 and estimates the cost of subsequent annual
services to be P100,000. Ruby expects a profit of 20% on subsequent services. Rodel paid the
annual fee for 2012 and Ruby rendered annual services for that year.

In its December 31, 2012 income statement, the realized franchise revenue to be reported
by Ruby is: P50,000
Upon receipt of Continuing Franchise Fee for 2011:

Cash 50,000
Revenue from CFF 25,000
Unearned CFF (50% x P50,000) 25,000

CFF (P50,000 x 4) P200,000


Less: Costs 100,000 50%
Profit P .

When costs of CFF had been rendered (with 20% markup):


Unearned CFF 25,000
Revenue from CFF 25,000

Cost of CFF [P25,000 – (P25,000 x .80]) 20,000


Cash or Accounts payable, etc. 20,000
Unearned CFF P25,000 100%
Costs of CFF 20,000 80%
Profit P . 20%

Therefore, at the end of 2012, the realized revenue amounted to P50,000:

Revenue from CFF (upon receipt of cash) P25,000


Revenue from CFF (when continuing services
had been rendered) 25,000
Total realized franchise revenue P50,000

9. Nena’s Lechon, Inc. franchiser, entered into franchise agreement with Aling Nena, franchisee.
On Marche 31, 2014. The total franchise fee is P500,000, of which P100,000 is payable upon
signing and the balance in four equal annual installments. The downpayment is refundable n the
event the franchisor fails to render services and none thus far had been rendered.

When Nena’s prepares its financial statements on March 31, 2014, the franchise fee
revenue to be reported is: P 0

The franchise fee revenue should be zero, since, no substantial performance of service
had been performed (and the downpayment is still refundable).

10. Ferragamo’s entered into a franchise agreement with Rusty. As per agreement on July 1, 2014,
Rusty is to pay Ferragamo an up-front franchise fee of P1,000,000 and subsequent annual
franchise fees of P50,000 over the next four years. Cost of initial franchise services rendered by
Ferragamo’s during the year is P250,000 which is substantial, and is estimates the cost of
subsequent annual services to be P10,000. Rusty paid the annual franchises fee for 2015, and
Ferragamo’s rendered the services for the year. In its December 31, 2015 income statement,
the amount of realized franchise fee revenue to be reported by Ferragamo’s is: P50,000
In 2014, the initial franchise fee of P1,000,000 is recognized as revenue. Therefore, in
2015 the only amount of revenue to be recognized is the P50,000, the continuing franchisee
fee.

11. On May 31, 2013 , Kenny received P200,000 from Rogers representing the down payment on
the franchise agreement signed on that date. Rogers issued promissory notes for the balance of
P1,000,000, payable in four equal semi-annual installments. Franchise services are
substantially completed by Kenny on semi-annual installment due on November 20, 2013 at an
aggregate cost of P900,000. The first semi-annual installment due on November 30, 2013 was
appropriately paid by Rogers. Accordingly, Kenny uses the accrual method in recording
franchise revenue. In its December 31, 2013 financial statements, how much would Kenny
report as deferred franchise revenue for the year? P 0

Substantial performance of the services had been rendered, the period of refund has
been expired (problem is silent), and the collectability of the note is reasonably assured
(problem is silent). Therefore, the initial franchisee fee is considered revenue and no amount is
allotted to unearned franchisee revenue.

12. Amy, Inc. enters into an agreement with Ronald’s Co., clothing the later with full authority to
operate as its franchise for a period of ten years. An initial franchise fee of P275,000, among
others, was stipulated in the contract and was promptly paid during the year 2013.

Assuming that Amy was able to perform the initial services during 2013, what is the
franchise revenue to be recognized in its year-end income statement? P275,000

13. On September 31, 2013, Criselda’s Inc. received from Ambo P550,000 representing franchise
fee. Franchise services were immediately started by Criselda’s and these were completed on
October 31, 2013 at cost amounting to P330,000. The franchise fee revenue to be reported
by Criselda’s in its October 31, 2013 income statement is: P550,000

Substantial performance had been rendered in 2013 and the remaining conditions are
assumed to have been met. Therefore, P550,000 is recognized as franchise fee revenue. The
cost P330,000 is not deducted to compute the franchise fee revenue because the requirement
is franchise revenue and not gross profit or net income.

14. PJD Enterprises, a franchisor, charges franchisees a franchise fee of P500,000. Of this amount,
a nonrefundable P200,000 is paid upon the signing of the contract with the balance payable in
three equal installments after each year thereafter starting 2013. PJD will assist in locating a
suitable business site conduct a market study, oversee the construction of facilities, and provide
initial training for employees.

On October 1, 2013, PJD entered into a franchising agreement to cover an entirely new
untested area. By December 31, 2013, PJD had substantially completed and rendered
appropriate services at a total cost of P150,000 but, somehow, has raised some doubts on the
collectability of the balance of the franchise fee. In its 2013 income statement, PJD
Enterprises should recognize profit of: P350,000
Collection in 2013 P200,000
Multiplied by: Gross profit rate
[100% - (P150,000 / P500,000)] 70%
Realized gross profit in 2013 P140,000

15. On January 3, 2013, PP Services, Inc. signed an agreement authorizing CC Company to


operate as a franchisee over a 20-year period for an initial franchise fee of P50,000 received
when the agreement was signed. CC commenced operation on July 1, 2013, at which date all of
the initial services required of PP had been performed. The agreement also provides that CC
must pay annually to PP a continuing franchise fee equal to 5% of the revenue from the
franchise. CC’s franchise revenue for 2013 was P400,000. For the year ended December 31,
2013, how much should PP record as revenue from franchise fees in respect of CC’s
franchise? P70,000

Initial franchise fee P50,000


Continuing franchise fee (5% x P400,000) 20,000
Revenue from franchise P70,000

16. On December 31, 2013, RR Inc. authorized Fay to operate as a franchisee for an initial
franchise fee of P75,000. Of this amount, P30,000 was received upon signing the agreement,
and the balance, represented by a note, is due in three annual payments of P15,000 each,
beginning December 31, 2014. The present value of December 31, 2013 of the three annual
payments appropriately discounted is P36,000. According to the agreements, the nonrefundable
down payment represents a fair measure of the services already performed by RR, however,
substantial future services are required of RR. Collectability of the note is reasonably certain.
On December 31, 2013. RR should record unearned franchise fees in respect of the Fay
franchise of P36,000

The P30,000 down payment received by RR Inc. represents the earned franchise fee
revenue in 2013 since the problem already stated that the nonrefundable downpayment
represents a fair measure of the services already performed by RR, Inc. the present value of the
three annual payments to be received in the future of P36,000, should be reported as of
December 31, 2013 as unearned franchise fees:

Incidentally, the entry would be:

Cash 30,000
Notes Receivable 45,000
Unearned interest income 9,000
Franchise revenue 30,000
Unearned franchise revenue 36,000

17. Ruby Co. charges new franchisees an initial fee of P2,500,000. Of this amount, P1,000,000 is
payable in cash when the agreement is signed, and the remainder is to be paid in three equal
annual installments which are evidenced by interest-bearing promissory notes. In consideration
therefore, Ruby Co. will assist in locating the business site, conduct a market study to estimate
earnings potentials, supervise construction of a building, and provide initial training to
employees.
On December 3, 2013, Ruby Co. entered into a franchising agreement with Jade, Inc. by the
end of the year, Ruby Co. has completed about 25% of the initial services at a cost of P150,000
and it has ascertained that collection of the notes is reasonably assured. For 2013, Ruby Co.
should recognized franchise revenue of: P 0

Since 25% completion of initial services is not substantial, therefore, the P2,500,000
initial franchise fee lacks one condition in order to be recognized as revenue.

18. Shake, Inc. granted a franchise to Drake for the greenbelt area. Drake was to pay a franchise
fee of P100,000 payable in five equal annual installments starting with the payment upon
signing of the agreement. The franchisee was to pay monthly 1% of gross sales of the
preceding month. Should the operation of the outlet prove to be unprofitable in the first year of
operations the franchise fee may be cancelled with whatever obligation owing Shake, Inc. in
connection with the P100,000 franchise fee, waived.

On the same year of granting the initial franchise fee, the first year operation generated a gross
sales of P500,000 which is considered to be profitable operations. For the first year, Shake,
Inc. earned franchise fee of: P105,000

Initial franchise fee P100,000


Continuing franchise fee (1% x P500,000) 5,000
P105,000

19. Tony awarded its Cebu franchise to Jara Co. for a total fee of P100,000. Of the said amount,
P50,000 was payable upon the signing of the agreement and the balance in two equal annual
payments. The contact provided that in the event the first year would result in an operating loss,
the franchising agreement may be cancelled. No services had so far been rendered. The entry
to record the granting of the Cebu franchise by Tony to Jara Co was as follows:

Obviously, at the time of signing the contract, substantial performance of services has
not yet been rendered, unless otherwise stated. So the P100,000 initial franchise fee is not a
revenue but unearned franchise revenue.

20. Swabe, Inc. charges an initial franchise fee of P500,000 for the right to operate as a franchise of
Swabe. Of this amount, P100,000 is payable when the agreement was signed and the balance
is payable in a noninterest bearing note in five annual payments of P80,000 each. In return for
the initial franchise fee, the franchisor will help locate the site, negotiate the lease or purchase of
the site, supervise the construction activity, and provide the book keeping services. The credit
rating of the franchise indicates that money can be borrowed at 8%. The present value of an
ordinary annuity of five annual receipts of P80,000 each discounted at 8% is P319,416.80. the
discount represents the interest revenue to be accrued by the franchisor over the payment
period.

If the probability of refunding the initial franchise fee is extremely low, the amount of
future services to be provided to the franchisee is minimal, collectability of the note is
reasonably assured and substantial performance has accrued: EARNED P419,416.80
Cash P100,000.00
Note at present value 319,416.80
P419,316.80

CONSTRUCTION ACCOUNTING

1. Jenny Construction Co. has two projects for which it reported, as of December 31, 2013 the
following information:

In thousand pesos:
Proj. A Proj.B
Contract price P4,800 P860
2011: Cost incurred P3,400
Percent completed 75%
2012: Cost incurred P1,250 P140
Percent completed 25% 15%

Using percentage-of-completion method of revenue recognition, gross profit on ProjectA


to be recognized in 2013 would be: P200,000

Contract Price P4,800,000


Multiplied by: Percentage-of-completion – output 75%
Recognized Revenue to date P3,600,000
Less: Recognized Revenue in prior year 0
Recognized Revenue in current year, 2013 P3,600,000
Less: Cost incurred each year (for 2013 only) 3,400,000
Recognized gross profit in current year, 2013 P 200,000

2. Lovely Construction Co. was engaged on October 1, 2013 to construct a building for a contract
price of P8,400,000 payable in 5 installments. 1/5 of the contract price was to be paid upod
completion of each quarter of the work, the final payment being due within 10 days after
acceptance of the completed project.

By December 29, 2013, 3/4 of the building had been completed whereupon the third billing was
made in accordance with the term of the contract (cash had been received on the previous
billings). During 2013, a total of P4,200,000 had been disbursed by Lovely for costs incurred
and, at year-end, outstanding accounts payable for materials purchases totaled P1,000,000.
Lovely expected that an additional P1,800,000 would be required to complete the project.

Using the percentage-of-completion method on an output basis proportion method, the


gross profit to be recognized in the 2013 income state would be: P1,050,000

Contract Price P8,400,000


Less: Total Estimated Costs:
Costs Incurred to date (P4,200,000 +
P1,000,000) P5,200,000
Add: Estimated Costs to complete 1,800,000 7,000,000
Estimated gross profit P1,400,000
Multiplied by: Percentage-of-completion – output 3/4
Recognized gross profit to date P1,050,000
Less: Recognized gross profit in prior year -0- .
Recognized gross profit in current year, 2013 P1,050,000

3. Using the same information in No.2, percentage-of-completion output method-actual costs


approach, the gross profit to be recognized in 2013 income statement would be:
P1,100,000

Contract Price P8,400,000


Multiplied by: Percentage-of-completion – output 3/4
Recognized revenue to date P6,300,000
Less: Recognized Revenue in prior year -0- .
Recognized revenue in current year, 2013 P6,300,000
Less: Cost incurred each year (for 2013) 5,200,000
Recognized gross profit in current year, 2013 P1,100,000

4. On September 30, 2013, Jojo Co, Inc was awarded the contract to build a 1,000-room hotel for
120 million. Among others, the parties agreed the following:

 10% mobilization fee (deductible from “final billing”) payable within ten days from the
signing of the contract;
 Retention of 10% on all billings (to be paid with the final billing, upon completion and
acceptance of the project); and
 Progress billings are to be paid within 2 weeks upon acceptance.

By the end of 2013, the company had presented one progress billing, corresponding to 10%
completion, which was evaluated and accepted by the client on December 29, 2013 for payment
in January of the next year.

In 2013, assuming use of the percentage-of-completion method of accounting. Jojo Co.,


Inc. received cash a total fee of: P12,000,000

Contract Price P120,000,000


Multiplied by: Mobilization Fee 10%
Cash Received P 12,000,000

5. Cocaine Builders, Inc. employs the cost-to-cost method in determining the percentage of
completion for revenue recognition. the company’s records show the following information on a
recently completed project for a contract price of P5,000,000.

2010 2011 2012


Cost incurred to date P900,000 P2,550,000 ?
Gross profit (loss) 100,000 350,000 (50,000)
Compute the (1) estimated costs to complete the project at December 31, 2011 and (2) the
actual cost incurred during the year 2012 (1) 1,700,000; (2) 2,050,000

Estimated cost to complete the project at December 31, 2012:

Recognized gross profit in 2011 P 350,000


Add: Recognized gross profit in prior year – 2010 100,000
Recognized gross profit to date in 2011 (cumulative) P 450,000
Add: Costs incurred to date in 2011 (cumulative) 2,550,000
Recognized revenue to date (cumulative) P3,000,000
Divided by: Contract Price P5,000,000
Percentage of completion 60%

Therefore, the traditional formula to compute the % of completion would be as follows:

Cost Incurred to date - 2011


% of Completion = -------------------------------------------------
Total Estimated Cost – 2011

P2,550,000
60% = ---------------------
T. E. C.

T. E C = P4,250,000

Total Estimated Costs – 2011 P4,250,000


Less: Cost to date – 2011 2,550,000
Cost incurred in 2011 P1,700,000

Cost incurred in 2012:


Contract Price P5,000,000
Less: Total Estimated gross profit
(P100,000 + P350,000 – P50,000) 400,000
Total estimated cost (or total actual cost) P4,600,000
Less: Cost incurred to date – 2011 (cumulative) 2,550,000
Cost incurred in 2011 P2,050,000

6. In 2011, AJD Construction Co. was contracted to build Village Company’s private road network
for P100 million. The project was estimated to be completed in two years, and the contract
provided for:

 5% mobilization fee (to be conducted from the last billing) payable within 15 days after
the signing of the contract,
 10% retention provision on all billings, and
 Payment of progress billings within 10 days from acceptance.
AJD, which uses the percentage-of completion method of accounting, estimated a 25% gross
margin on the project. By the end of 2011, AJD had presented progress billings corresponding
to 50% completion all of the progress billings presented in 2011 was accepted, except the last
one for 10% which was accepted on January 2, 1012 with the exception of one bill for 8% which
was due on January 7, 2012, all of the billings accepted in 2011 were settled payments made by
Village Company in 2011 amounted to: P33,800,000

Mobilization Fee: P100,000,000 x 5% P5,000,000


Collections on Progress Billings:
Contract Price P100,000,000
Multiplied by: Progress Billings
(net of late billings of 10% and 8%) 32%
Billings, net P 32,000,000
Multiplied by: Collection % (Net of 10%
contract retention) 90% 28,800,000
Collections / Payments P33,800,000

8. In 2011, Joey Builders was contracted to build the private road network of Althea Subdivision for
P100 million. The project was expected to be finished in 2 years, and the contract provided for:

o 5% mobilization fee (to be deducted from the last billing), payable within 15 days from
the contract signing.
o A retention provision of 10% on all billings, payable with the final bill after the completed
project is accepted.
o Payment of progress billings within 7 days from acceptance.

Joey Builders, which uses the percentage-of-completion method of accounting for income,
estimated a 25% gross margin on the project. By the end of the year, Joey Builders had
presented progress billings to Althea corresponding to 50% completion. Althea accepted all the
bills presented, except one for 10% which was accepted o January 5 of next year. With the
exception of the second to the last billing for 8% which was due January 3 of next year, all
accepted billings were settled.

In 2011, Joey Builders realized gross profit from the project the amount of: P12,500,000

Contract Price P100,000,000


Multiplied by: Gross Profit Rate 25%
Estimated gross profit at the entire contract P 25,000,000
Multiplied by: Percentage of Completion for first year 50%
Realized Gross Profit in current year P 12,500,000

9. Dasma Corporation entered into a construction agreement in 2012 that called for a contract
price of P9,600,000. At the beginning of 2013, a change order increased the initial contract price
by P480,000. The company uses the percentage-of completion basis of revenue recognition. in
relation to the project, the following are obtained;

2012 2013
Cost incurred to date P4,920,000 P8,640,000
Estimated costs to complete 4,920,000 2,160,000
Billings made 5,280,000 8,520,000
Collection made 4,380,000 7,500,000

What gross profit (loss) should Dasma Corp. recognize in 2013?

2011 2012
Contract price P9,600,000 P10,080,000

Less: Total Estimated Costs:


Costs incurred to date P4,920,000 P 8,640,000
Add: Estimated costs to complete 4,920,000 2,160,000
Total estimated costs P9,840,000 P10,800,000
Estimated gross profit P(240,000) P( 720,000)
Multiplied by: percentage of completion 100% 100%
Recognized gross profit to date P(240,000) P( 720,000)
Less: Recognized gross profit in prior year _________ ( 240,000)
Recognized gross profit each year P(240,000) P( 480,000)

10. During 2011, Rizza started work on a P3,000,000 fixed-price construction contract. Any costs
incurred are expected to be recoverable. The accounting records disclosed the following date
for the year ended December 31, 2011;

Costs incurred P 930,000


Estimated cost to complete 2,170,000
Progress billings 1,100,000

How much should Rizza have recognized in 2011?

Contract Price P3,000,000


Less: Total Estimated Costs P 930,000
Cost incurred to date 2,170,000 3,100,000
Add: Estimated costs to complete P( 100,000)

11. Daryl Construction Company has consistently used the percentage-of-completion method of
recognizing income. During 2011, Daryl entered into a fixed-price contract to construct an office
building for P10,000,000. Information relating to the contract is as follows:

12/31/2011 12/31/2012
Percentage of completion 20% 60%
Estimated total cost at completion P7,500,000 P8,000,000
Income recognized (cumulative) 500,000 1,200,000

Contract costs incurred during 2012 were P3,300,000

Cost incurred to date, 12/31/2011


P7,500,000 x 20% P1,500,000
Less: Cost incurred to date, 12/31/2012:
P8,000,000 x 60% 4,800,000
Cost incurred during 2012 P3,300,000

12. The October 1, 2010, Neo Corp. enters a contract to build a sport arena which it estimated
would cost P3,120,000. Neo bills its clients at cost plus 20% and recognized construction
revenue on a percentage-of-completion basis. Data on this project for 2010, 2011 and 2012
follow:

Costs Est’d Costs


Incurred to Complete
2010 P 546,000 P2,054,000
2011 998,400 1,315,600
2012 1,575,600 -

Neo Corps.’ Gross profit on the project for 2012 is P146.640

2011 2012
Contract Price (P3,120,000 x 120%) P3,744,000 P3,744,000

Less: Total Estimated Costs:


Cost incurred each year P 998,400 P1,575,600
Add: Costs incurred in prior year 546,000 1,544,400
Costs incurred to date P1,544,400 P3,120,000
Add: Estimated costs to complete 1,315,600 _________
Total estimated costs P2,860,000 P3,120,000
Estimated gross profit 884,000 P 624,000
Multiplied by: percentage of completion 54% 100%
Recognized gross profit to date P 477,360 P 624,000
Less: Recognized gross profit in prior year - . 477,360
Recognized gross profit each year P 477,360 P 146,640

13. The Matibay Construction Corporation uses the percentage-of-completion method of


recognizing income from long-term construction contracts. In 2010 Matibay entered into a fixed-
price contract to construct a bridge for P30,000,000. Estimated costs to complete the
construction and contract cost incurred up to 2010 were as follows:

Cumulative Estimated costs


costs incurred to complete
As of December 31, 2010 P 2,000,000 P16,000,000
As of December 31, 2011 11,000,000 11,000,000
As of December 31, 2012 20,000,000 4,000,000

What is the percentage of completion during the year 2012? 33 1/3%

Costs incurred to date P11,000,000 P20,000,000


Add: Estimated costs to complete 11,000,000 4,000,000
Total estimated cost P22,000,000 P24,000,000
Percentage of completion 50% 83 ½ %
Cost incurred to date / TEC

Percentage of completion as of 12/31/2012 83 ½ %


Less: Percentage of completion as of 12/31/2011 50%
Percentage of completion in 2012 33 1/3 %

14. THE Kirby Construction Company has consistently used the cost recovery method –
construction accounting method of recognizing income (zero-profit approach)

In 2011, it began a construction project to erect a building for P3,000,000. The project was
completed during 2012, under this method, the accounting records disclosed the following:
(ANY COST INCURRD ARE EXPECTED TO BE RECOVERABLE)

Progress billing during the year P1,100,000 P1,900,000


Costs incurred during the year 900,000 1,800,000
Collection on billings during the year 900,000 2,100,000
Estimated costs to complete the project 1,800,000 -

The company should recognized revenue for the year amount to;

Recognized Revenue P900,000 P2,100,000


Less: Cost of long-term construction contract 900,000 1,800,000
Recognized gross profit each year P 0 P 300,000

15. The following date relate to a construction job started by Jay Company during 2011:

Total contract price P100,000


Actual costs during 2011 20,000
Estimated remaining costs 40,000
Billed to customer during 2011 30,000
Received from customer during 2011 20,000

Any costs incurred are expected to be recoverable. Under the cost recovery method-
construction accounting (zerp-profit approach). What amount should Jay Company
recognize as gross profit for 2011: P0

Recognized Revenue in 2011 P20,000


Less; Cost of long-term construction (cost incurred in 2011) 20,000
Recognized gross profit in current year – 2011 0

16. During 2011, Mitch Corporation started a construction job with a total contract price of
P600,000. Any costs incurred are expected to be recoverable. The job was completed on
December 15, 2012. Additional data are as follows;
2011 2012
Actual costs incurred P225,000 255,000
Estimated remaining costs 225,000
Billed to customer 240,000 360,000
Received from customer 225,000 375,000

Under the cost recovery method of construction accounting, what amount should Mitch
recognize as gross profit for 2011 and 2012/

2011 2012
Recognized revenue P225,000 P375,000
Less: Cost of long-term construction contract 225,000 255,000
Recognized gross profit in current year P 0 P120,000

17. Aibee Construction Company has consistently used the percentage-of-completion method. On
January 10, 2011, Aibee began work on a P6,000,000 construction contract. At the inception
date, the estimated cost of construction was P4,500,000. The following data relate to the
progress of the contract:

Income recognized at 12/31/2011 600,000


Cost incurred 1/10/2011 through 12/31/2012 3,600,000
Estimated cost to complete at 12/31/2012 1,200,000

How much income should Aibee recognize for the year ended December 31, 2012?
300,000

Contract Price 6,000,000

Less: Total Estimated Costs:


Costs incurred to 1/10/11 – 12/31/12 3,600,000
Add: Estimated costs to complete 1,200,000 4,800,000
Estimated gross profit 1,200,000
Multiplied by: percentage-of-completion 36/48
Recognized gross profit to date 900,000
Less: Recognized gross profit in prior year 600,000
Recognized gross profit each year – 2012 300,000

18. MM Construction Company has consistently used the percentage-of-completion method of


recognizing income. During 2011, MM started work on a P3,000,000 construction contract which
was completed in 2012. The accounting records provided the following data:

Progress billings 1,100,000 1,900,000


Costs incurred each year 900,000 1,800,000
Collections 700,000 2,300,000
Estimated cost to complete 1,800,000

How much income should MM have recognized in 2012? 200,000

Contract Price 3,000,000 3,000,000


Less: Total Estimated Costs:
Costs incurred each year 900,000 1,800,000
Add: Cost incurred each in prior years ________ 900,000
Cost incurred to date 900,000 2,700,000
Add: Estimated costs to complete 1,800,000 ________
Total estimated costs 2,700,000 2,700,000
Estimated gross profit 300,000 300,000
Multiplied by: percentage-of-completion 9/27 100%
Recognized gross profit to date 100,000 300,000
Less: Recognized gross profit in prior year ________ 100,000
Recognized gross profit each year 100,000 200,000

19. Atom Inc. has entered into a very profitable fixed price contract for constructing a high-rise
building over a period of three years. It incurs the following costs relating to the contract during
the first year:


 Cost of material = 2.5 million
 Site labor cost = 2.0 million
 Agreed administrative costs as per contract to be reimbursed by the customer = 1 million
 Depreciation of the plant used for the construction = 0.05 million
 Marketing costs for selling apartments, when they are ready = 1,0 million

Total estimated cost of the project = 18 million

The percentage of completion of this contract at the year-end is: 33 1/3% (=6.0/18.0)

Costs incurred to date:


Materials 2,500,000
Direct labor 2,000,000
Overhead
Administrative costs reimbursable 1,000,000
Depreciation during construction 500,000 6,000,000
Divided by: Total estimated costs 18,000,000
Percentage of completion 33 1/3 %

20. IOM Builders, Inc. has consistently used the percentage-of-completion method of accounting for
construction-type contracts. During 2011, IOM started work on a P9,000,000 fixed-price
construction that was completed in 2012, IOM accounting records disclosed the following:

12/31/2011 12/31/2012
Cumulative contract costs incurred 3,900,000 6,300,000
Estimated total costs at completion 7,800,000 8,100,000

How much income would IOM have recognized on this contract for the year ended
December 31, 2012? 100,000
Contract Price 9,000,000 9,000,000
Less: Total Estimated Costs: 7,800,000 8,100,000
Estimated gross profit 1,200,000 900,000
Multiplied by: percentage-of-completion 39/78 63/81
Recognized gross profit to date 600,000 700,000
Less: Recognized gross profit in prior year ________ 600,000
Recognized gross profit each year – 2012 600,000 100,000

CORPORATE LIQUIDATION

1. Philippine National Bank holds a P500,000 note secured by a building owned by Luigi
Software, which has a filled for bankruptcy. If the property has a book value of P600,000
and a fair market value of P450,000, what is the best way to describe the notes held by
Philippine National Bank? The bank has

a. A secured claim of P500,000.


b. An unsecured claim of P500,000.
c. A secured claim of P450,000 and unsecured claim of P50,000.
d. A secured claim of P50,000 and unsecured claim of P50,000.

Answer: c
The 500,000 notes payable to PNB is considered as partially secured liabilities
wherein a property with a fair market value of P450,000 is used as collateral.
Therefore, PNB is secured to receive P450,000 because of the property while the
balance of P50,000 of the notes is unsecured.

2. A and B Inc. owes the Xylo Corporation P60,000 on account, which is secured by
accounts receivable with a book value of P50,000. The unsecured portion is considered
a claim under the bankruptcy law, A and B has filed for bankruptcy. Its statement of
affairs lists the accounts receivable securing the Xylo account with an estimated
realizable value of P45,000. If the dividend to general unsecured creditors is 80%, how
much can Xylo expect to receive?

a. P60,000 c. P57,000
b. 58,000 d. 48,000

Answer: c
The P60,000 owes to Xylo Corp. is considered a partially secured liabilities.
Accounts receivable with a realizable value of P45,000 is pledge to secure the
liability. Therefore, the estimated amount to be paid to Xylo Corp. would be as
follows:

Accounts Receivable at net realizable value P45,000


Add: Portion of free assets used to pay the unsecured
amount: (P60,000 – P45,000) x .80 12,000
Estimated amount to be paid to partially secured Liabilities P57,000

3. P Corporation is a parent, having purchased 60% of S Company’s common stock at


par value for P600,000. S Company is in financial difficulty. The parent granted an
unsecured loan of P200,000 to the subsidiary. An accounting statement of affairs for S
Company shows a dividend of 30%. P Corporation can expect to receive on the loan of
appropriately:

a. P120,000 c. P36,000
b. 60,000 d. 0

Answer: b
Since the P Corp. expect to recover P.30 for every P1 liability. Therefore, the
unsecured liability of S Company that would be paid were as follows:

Unsecured loan P200,000


Multiplied by: Expected recovery per peso of
unsecured creditors 30%
P 60,000

4. P Corporation is a parent, having purchased 60% of S Company’s common stock at par


value for P 600,000. S Company is in financial difficulty. The parent granted as
unsecured loan of P 200,000 to the subsidiary. An accounting statement of affairs for S
Company shows a dividend of 30%. P Corporation can expect to receive payment for its
investment in S Company of approximately:

a. P600,000 c. P108,000
b. 180,000 d. 0

Answer: d
This problem is similar to No. 3 except that the question involves payment for its
investment in S Company. Remember that receivables and payables transacted
between parent and subsidiary still exist on their separate balance sheet. So,
when collection or payment is made, it will still be journalized in the usual
manner, like an ordinary collection or payment of an account which does not
affect at all investment in subsidiary account.
5. Kent Inc. has forced into bankruptcy and has begun to liquidate. Unsecured claims will
be paid at the rate of 40 cents on the peso. Apex Co. holds a non-interest bearing note
receivable from Kent in the amount of P100,000, collateralized by machinery with a
liquidation value of P25,000. The total amount to be realized by Apex on this note
receivable is:

a. P25,000 c. P55,000
b. 40,000 d. 65,000

Answer: c
Apex Co. has a secured claim for the P25,000 liquidation value of the machinery.
The remaining P75,000 (P100,000 note – P25,000) is an unsecured claim. Given
that unsecured claims will be paid at the rate of P.40 cents on the peso,
therefore, Apex will receive:

Machinery at liquidation value P25,000


Add: Portion of free assets used to pay the unsecured
amount: (P100,000 – P25,000) x 40% 30,000
P55,000

6. Seco Corp. was forced into bankruptcy and is in the process of liquidating assets and
paying claims. Unsecured claims will be paid at the rate of forty cents on the peso. Hale
holds a P30,000 non-interest bearing note receivable from Seco collateralized by an
asset with a book value of P35,000 and a liquidation value of P5,000. The amount to be
realized by Hale on this note is

a. P5,000 c. P15,000
b. 12,000 d. 17,000

Answer: c
Claims of secured creditors be satisfied before any unsecured claims are paid.
Hale is a secured creditor in the amount of P5,000 (the liquidation value of the
collateral). The remainder of Hale’s claim (P30,000 – P5,000 = P25,000) is an
unsecured claim, because it is not secured by any collateral. Therefore, Hale, will
receive a total of P15,000 on this note:

Asset at liquidation value P 5,000


Add: Portion free assets is used to pay the unsecured
amount: (P30,000 – P5,000) x .40 10,000
P 15,000
7. Blueprint Inc. signed a note payable to its bank for P10,000. Accrued interest on the
note on February 28, 2004 amounts to P250. The note is secured by inventory with a
book value of P12,000. The inventory is sold for P8,000 and unsecured creditors
receive 30 percent of their claims. The bank should receive the following amount in
settlement of the note and interest:

a. P10,250 c. P8,675
b. 10,000 d. 8,000

Answer: c
Inventory at selling price P 8,000
Add: Portion free assets used to pay the unsecured
amount (P10,250 – P8,000) x 30% 675
P 8,675

8. The trust for Ardolio, Inc. prepares a statement of affairs which shows that unsecured
creditors whose claims total P60,000 may expect to receive approximately P36,000 if
assets are sold for the benefit of creditors.

 Michael is an employee who is owed P750.


 Meldcan holds a note for P1,000 on which interest of P50 is accrued;
nothing has been pledged on the note.
 Compboy holds a note of P6,000 on which interest of P300 is accrued;
securities with a book value of P6,500 and a present market value P5,000
are pledged on the note.
 Serpor holds a note for P2,500 on which interest of P150 is accrued
property with a book value of P2,000 and a present market value of
P3,000 is pledged on the note.

How much may each of the following creditors hope to receive?

Michael Meldcan Compboy Serpor


a. P 0 P 0 P 0 P 0
b. 90 0 6,300 2,390
c. 350 1,050 5,780 0
d. 750 630 5,780 2,650

Answer: d
Michael’s salary is an unsecured with priority, therefore he receives the full
amount. Meldcan: P1,050 x (P36,000 / P60,000)= P630
Compboy: P5,000 + (P6,300 – P5,000) x 60% = P5,780
Serpor: Fully secured creditor, receive P2,650 (P2,500 + P150)
9. Erap Co. filed a voluntary bankruptcy petition on August 15, 2008, and the statement of
affairs reflects the following amounts:

Book Value Estimated


Current Value
Assets:
Assets pledged with fully secured creditors P 300,000 P 370,000
Assets pledged with partially secured
Creditors 180,000 120,000
Free assets 420,000 320,000
P 900,000 P 810,000

Liabilities:
Liabilities with priority P70,000
Fully secured creditors 260,000
Partially secured creditors 200,000
Unsecured creditors 540,000
P1,070,000

Assume that the assets are converted to cash at the estimated current values and the
business is liquidated. What amount of cash will be available to pay unsecured non-
priority claims?

a. P240,000 c. P320,000
b. 280,000 d. 360,000

Answer: d
The total amount to pay all unsecured claims, including priority claims, is the
cash obtained from free assets (P320,000) and any excess cash available from
free assets pledged with fully secured creditors after they are used to satisfy
those claims (P370,000 – P260,000 = P110,000).
Therefore, the amount of cash to pay unsecured non-priority claims:

Assets pledged to fully secured creditors,


at current value P370,000
Less: Fully secured creditors 260,000
Excess cash from assets pledged to fully
secured creditors P110,000
Add: Free assets, at current value 320,000
Total free assets P430,000
Less: Liabilities with priority 70,000
Net free assets P360,000
10. Zamora and Co., Inc. purchased a Cadillac automobile with little cash down and signed
a note, secured by the Cadillac, for 48 easy monthly payments. When the company files
for bankruptcy, the balance due on the Cadillac amount to P6,000,000. The car has a
book value of P8,000,000 and a net realizable value of P4,000,000. The unsecured
creditors of Zamora and co. can expect to receive 50 percent of their claims. In the
liquidation, the bank that holds the note on the Cadillac should receive:

a. P6,000,000 c. P4,000,000
b. 5,000,000 d. 3,000,000

Answer: b
Car-cadillac, at net realizable value P4,000,000
Add: Portion of free assets used to pay unsecured
Amount; (P6,000,000 – P4,000,000) x 50% 1,000,000
P5,000,000

11. The following data are provided by the Troubled Company:

Assets at book value P150,000


Assets at net realizable value 105,000
Liabilities at book value:
Fully secured mortgage 60,000
Unsecured accounts and notes payable 70,000
Unsecured liabilities:
Interest on bank notes 500
Estimated cost of administering estate 6,000

The count has appointed a trustee to liquidate the company.

The journal entry made by trustee to record the assets and liabilities should include an
estate deficit of:

a. P31,500 c. P25,500
b. 31,000 d. 25,000

Answer: c
To compute the estate deficit before the actual realization and liquidation is
simply to formulate the basic accounting equation, i.e. Assets = Liabilities +
Stockholder’s equity. Therefore:

Assets, at net realizable value P105,000


Less: Liabilities
Per books P130,000
Add: Unrecorded interest 500 130,500
Estate (deficit) equity before realization
and liquidation P( 25,500)

12. Using the same information in Number 11, the statement of affairs prepared by the
trustee at this time should include an estimated deficiency to unsecured creditors of:

a. P45,000 c. P31,500
b. 39,000 d. 25,500

Answer: c

Total assets at net realizable value P105,000


Less: Fully secured liabilities 60,000
Total free assets P 45,000
Less: Unsecured creditors with priority –
Administrative expenses 6,000
Net free assets P 39,000
Less: Unsecured creditors without priority
Unsecured accounts and notes payable P70,000
Interest on bank notes 500 70,500
Estimated deficiency to unsecured creditors P(31,500)

13. Nah Lugi Corporation is in bankruptcy and is being liquidated by a court-appointed


trustee. The financial report that follow was prepared by the trustee just before the final
cash distribution;

Assets:
Cash P100,000

Approved claims:
Mortgage payable (secured by property
that was sold for P50,000) P 80,000
Accounts payable, unsecured 50,000
Administrative expenses payable,
Unsecured 8,000
Salaries payable, unsecured 2,000
P140,000

The administrative expenses are for trustees and other costs of administering the debtor
corporation’s estate.
How should the P100,000 be distributed to the following creditors?

Unsecured Creditors Partially Secured Unsecured Creditors


With Priority CreditorsWithout Priority

a. P - P 80,000 P20,000
b. 10,000 80,000 10,000
c. 5,000 65,000 25,000
d. 10,000 65,000 25,000

Answer: d
Cash available P100,000
Less: Mortgage payable secured by property 50,000
Amount available to unsecured creditors P 50,000
Less: Unsecured creditors with priority
Administrative expenses P 8,000
Salaries payable 2,000 10,000
Net free assets or amount available to
Unsecured creditors with priority P 40,000
Expected recovery percentage of unsecured creditors
P40,000 / (P80,000 – P50,000) + P50,000 P .50

Therefore, the cash is distributed as follows:


Unsecured creditors with priority P 10,000
Partially secured creditors:
Property at selling price P 50,000
Add: Portion of free assets used
to pay the unsecured amount
(P80,000 – P50,000) x 50% 15,000 65,000
Unsecured Creditors without priority
P50,000 x .50 25,000
P100,000

14. The following data were taken from the statement of affairs for Liquo Company:

Assets pledged for fully secured liabilities


(fair value, P75,000) P 90,000
Assets pledged to partially secured liabilities
(fair value, P52,000) 74,000
Free assets (fair value, P40,000) 70,000
Unsecured liabilities with priority 7,000
Fully secured liabilities 30,000
Partially secured liabilities 60,000
Unsecured liabilities without priority 112,000
Compute the: (1) total estimated deficiency to unsecured creditors, and (2) the
expected recovery per peso unsecured claims.
a. (1) 42,000; (2) P.65 c. (1) P 0; (2) P1.00
b. (1) 3,000; (2) P.98 d. (1) P42,000; (2) P .70

Answer: a

Estimated deficiency to unsecured creditors: P 75,000


Assets pledged to fully secured liabilities, at fair value 30,000
Less: Fully secured liabilities 45,000
Add: Free Assets, at fair value 40,000
Total free assets to unsecured liabilities P 85,000
Less: Unsecured liabilities with priority 7,000
Net free assets P 78,000
Less: Unsecured liabilities:
Partially secured liabilities P 60,000
Less: Assets pledged to partially
Secured liabilities, fair value 52,000 P 8,000
Unsecured liabilities without priority 112,000
Total unsecured liabilities P120,000
Estimated deficiency to unsecured liabilities P 42,000

15. Katherine, a CPA, has prepared a statement of affairs. Assets which there are no
claims or liens are expected to produce P70,000, which must be allocated to
unsecured claims of all classes totaling P105,000. The following are some of the
claims outstanding:

1. Accounting fees for Katherine, P1,500


2. An unrecorded note for P1,000, on which P60 of interest has accrued,
held by Angie.
3. A note for P3,000 secured by P4,000 receivables, estimated to be 60%
collectible held by Joy.
4. A P1,500 note, on which P30 of interest has accrued, held by Joyots.
Property with a book value of P1,000 and a market value of P1,800 is
pledged to gurantee payment of principal and interest.
5. Unpaid income taxes of P3,500.

Compute the estimated payment to partially secured creditors:


a. P1,060 c. P2,490
b. 1,950 d. 2,790

Answer: d
Total free assets P 70,000
Less: Unsecured Creditors with Priority:
Administrative expenses – accounting fees P1,500
Unpaid income taxes 3,500 5,000
Net free assets P 65,000
Total Unsecured Creditors without Priority:
Total Unsecured Claims of all classes P105,000
Less: Unsecured Creditors with Priority 5,000
Total Unsecured Creditors without Priority P100,000
% of Recovery; P65,000 / P100,000 = 65%
Estimated payment to partially secured creditors:
Realizable value of A/R (60% x P4,000) P 2,400
Add: Unsecured Portion: 65% (P3,000 – P2,400) 390
Total P 2,790

16. Palubog Co. is insolvent and its statement of affairs shows the following
information:

Estimated gains on realization of assets P1,440,000


Estimated losses on realization of assets 2,000,000
Additional assets 1,280,000
Additional liabilities 960,000
Capital Stock 2,000,000
Deficit 1,200,000

The pro-rate payment on the peso to stockholders (estimated amount ot be


recovered by stockholders) is:

a. P.30 c. P .57
b. .43 d. .70

Answer: d
Estimated losses on realization of assets
P2,000,000
Less: Estimated gains on realization of assets P1,440,000
Additional assets 1,280,000 2,720,000
Estimated net (gain) or loss in assets realization P
(720,000)
Add: Additional liabilities
960,000
Estimated net (gain) or loss P
240,000
Less: Stockholder’s equity:
Capital Stock P2,000,000
Deficit 1,200,000 800,000
Estimated amount to be recovered by stockholders P
560,000
Therefore, the pro-rate payment on the peso is:
Estimated amount to be recovered by stockholders=P560,000 = P.70
Stockholder’s Equity P800,000
17. Zero Na Corp. has been undergoing liquidation since January 1. As of March 31,
its condensed statement of realization and liquidation is presented below:

Assets:
Assets to be realized P1,375,000
Assets acquired 750,000
Assets realized 1,200,000
Assets not realized 1,375,000

Liabilities:
Liabilities liquidated P1,875,000
Liabilities not liquidated 1,700,000
Liabilities to be liquidated 2,250,000
Liabilities assumed 1,625,000

Revenue and Expenses:


Supplementary Charges P3,325,000
Supplementary Credits 2,800,000

The net gain (loss) for the three-month period ending March 31 is:

a. P250,000 c. P425,000
b. (325,000) d. 750,000

Answer: c
Statement of Realization and Liquidation Credits:
Assets realized P1,200,000
Assets not realized 1,375,000
Liabilities to be liquidated 2,250,000
Supplementary credits* 2,800,000
Total credits P9,250,000

Statement of Realization and Liquidation Debits:


Assets to be realized P1,375,000
Assets acquired 750,000
Liabilities liquidated 1,875,000
Liabilities not liquidated 1,700,000
Supplementary charge** 3,125,000
Net gain for the three month period P 425,000

*Supplementary credits are revenue or income items such as sales, interest income etc.
**Supplementary debits are cost and expense items such as purchases, expenses, etc.
18. Using the same information on No. 17, compute the ending cash balance of cash
account assuming that common stock and deficit are P1,500,000 and P500,000,
respectively.

a. P425,000 c. P1,325,000
b. P575,000 d. P1,375,000

Answer: c
The solution simply utilize the basic accounting equation of Assets = Liabilities +
Stockholder’s Equity (SHE), thus:
Common Stock P1,500,000
Deficits (500,000)
Stockholder’s Equity (SHE) P1,000,000
Add: Liabilities not liquidated 1,700,000
Total Liabilities and SHE P2,700,000
Less: Assets not realized (or end) 1,375,000
Cash balance, ending P1,325,000

Items 19 to 20 are based on the following data:

19. The Palubog Company has decided to seek liquidation after previous
restructuring and quasi-reorganization attempts failed. The company has the
following condensed balance sheet as of May 1, 2011:

AssetsLiabilities and Stockholder’s Equity

Cash P 12,000Accrued Payroll P 40,000


Receivables (net) 280,000 Loans from officer 50,000
Inventory 70,000 Accounts Payable
60,000
Prepaid Expenses 1,000Equipment loan payable 360,000
Plant Assets 300,000 Business loan payable
180,000
Goodwill 39,000 Common Stock
60,000
Deficit (48,000)
Total P702,000 Total
P702,000

The equipment loan payable is secured by specific plant assets having a book
value of P300,000 and a realizable value of P350,000. Of the account payable,
P40,000 is secured by inventory which has a cost of P40,000 and a liquidation
value of P44,000. The balance of the inventory has a realizable value of
P32,000. Receivables with a book value and market value of P100,000 and
P80,000 respectively have been pledged as collateral on the business loan
payable. The balance of the receivables have a realizable value of P150,000.

Assuming trustee expenses of P12,000 in addition to recorded liabilities, which of


the remaining unsecured creditors has the next higher order of priority.

a. Accrued Payroll c. Loan from officer


b. Equipment loan payable d. Business loan payable

Answer: a
Claims of unsecured creditors must be satisfied to whatever extent possible in
the following order of priority:
1. Expenses to administer the estate
2. Unpaid salaries, etc.

20. The realizable value of assets pledged with fully secured creditors is:

a. P459,000 c. P40,000
b. 44,000 d. 489,000

Answer: b
Of all the assets listed only inventory is classified as an asset pledged to fully
secured creditors with a realizable value of P44,000 (book value of accounts
payable is P40,000).

CASH FLOW

1. In 2011, Art Company had the following financial data:

Cash revenue 8,000,000


Cash expenses 4,000,000
Depreciation expense 2,000,000
Income before income tax 2,000,000
Income tax expense 500,000
Net income 1,500,000

At the beginning of 2012, the entity purchased additional assets at a cost of P5,000,000
on cash basis. Each year, these assets provide additional cash revenue of P5,000,000
and incur cash expenses of P2,000,000. The assets have a 10-year life and the entity
uses the straight line depreciation for all assets. The existing assets produced the same
cash revenue and incur the same expenses as in 2011. The income tax is paid every
April 15 of each year.

What is the net cash provided by operating activities for the current year?
6,500,000

Revenue (8,000,000 + 5,000,000) 13,000,000


Expenses (4,000,000 + 2,000,000) ( 6,000,000)
Income tax for 2011 paid on April 15, 2012 ( 500,000)
Net cash provided by operating activities 6,500,000

2. The cash balance of Darwin Company on January 1, 2011 was P8,000,000. During
2011, the changes in certain accounts were:

Accounts receivable 2,000,000 increase


Inventory 1,500,000 decrease
Accounts payable 3,000,000 decrease

Total sales and cost of goods old were P30,000,000 and P20,000,000 respectively. All
sales and purchases were made on credit. Various expenses of P5,000,000 were paid
in cash. There were no other pertinent transactions.

What is the cash balance on December 31, 2011? 9,500,000

Cash balance – January 1 8,000,000


Increase in accounts receivable ( 2,000,000)
Decrease in inventory 1,500,000
Decrease in accounts payable ( 3,000,000)
Sales 30,000,000
Cost of goods sold (20,000,000)
Expenses ( 5,000,000)
Cash balance – December 31 9,500,000

3. BUMPER Company’s statement of cash flows for the current year shows cash flow from
operations of P1,840,000.

Depreciation expense 400,000


Accounts receivable increase 120,000
Inventory decrease 280,000
Accounts payable decrease 80,000

What is the net income for the current year? 1,360,000


Net income 1,360,000
Depreciation 400,000
Accounts receivable increase ( 120,000)
Inventory decrease 280,000
Accounts payable decrease ( 80,000)
Cash flow from operations 1,840,000

4. Claire Company reported interest expense in 2011 and 2010 of P1,500,000 and
P1,200,000, respectively. The balance in accrued interest payable at the end of 2011,
2010 and 2009 was P600,000, P700,000 and P500,000, respectively. In addition, a note
to Claire Company’s 2011 financial statements included the following:

Interest costs related to construction in progress are capitalized as incurred. The entity
capitalized P300,000 and P250,000 of interest costs during the year 2011 and 2010,
respectively.

What amount of interest was paid in 2011 net of the amount capitalized as
construction in progress? 1,600,000

Interest expense in 2011 1,500,000


Accrued interest payable - December 31, 2011 ( 600,000)
Accrued interest payable – December 31, 2010 700,000
Interest paid in 2011 1,600,000

5. Lance Company provided the following for the current year:

Net income 6,000,000


Noncash adjustments:
Depreciation 900,000
Increase in accounts receivable ( 500,000)
Decrease in inventory 4,000,000
Decrease in accounts payable (1,200,000)
Net cash flow from operating activities 9,200,000

Lance reported revenue from customers of P7,500,000 for the current year. What
amount of cash was received from the customers? 7,000,000

Sales 7,500,000
Increase in accounts receivable ( 500,000)
Cash received from customer 7,000,000
6. Sarah Company reported bonds payable of P4,700,000 on December 31, 2010 and
P5,000,000 on December 31, 2011. During 2011, Sarah issued P2,000,000 of bonds
payable in exchange for equipment. There was no amortization of premium or discount
during the year.

What is the payment for redemption of bonds payable? 1,700,000

Bonds payable – December 31, 2010 4,700,000


Bonds payable issued in 2011 2,000,000
Bonds payable – December 31, 2011 (5,000,000)
Cash paid for bonds redemption in 2011 1,700,000

7. Cloudy Company provided the following information:

2011 2010
Retained earnings 3,000,000 2,500,000
Dividend payable 1,200,000 1,800,000
Net income 2,000,000

What amount was paid for dividends during the current year? 2,100,000

Retained earnings – 2010 2,500,000


Net income for 2011 2,000,000
Total 4,500,000
Retained earnings – 2011 (3,000,000)
Dividends declared in 2011 1,500,000
Dividends payable – 2010 1,800,000
Dividends payable – 2011 (1,200,000)
Cash paid for dividends in 2011 2,100,000

8. Sam Company provided the following for the current year:

Purchase of real estate for cash 5,500,000


Sale of investment securities for cash 5,000,000
Dividend paid 6,000,000
Issuance of ordinary shares for cash 2,500,000
Purchase of patent for cash 1,250,000
Payment of bank loan 1,500,000
Increase in customer’s deposit 200,000
Issuance of bonds payable for cash 3,000,000

What is the net cash provided by financing activities? 3,500,000


Cash borrowed from bank 5,500,000
Dividend paid (6,000,000)
Issuance of ordinary shares for cash 2,500,000
Payment of bank loan (1,500,000)
Issuance of bonds payable 3,000,000
Net cash provided – financing 3,500,000

9. What is the net cash used in investing activities? (1,750,000)

Purchase of real estate (5,500,000)


Sale of investment securities 5,000,000
Purchase of patent for cash (1,250,000)
Net cash provided – investing (1,750,000)

10. Dan Company collected the following data for the current year:

Gain on sale of equipment 60,000


Proceeds from sale of equipment 100,000
Purchase of bond investment (FV P2,000,000) 1,800,000
Amortization of bond discount 20,000
Dividend declared 450,000
Dividend paid 380,000
Proceeds from sale of treasury shares (cost P650,000) 750,000

What is the net cash provided by financing activities? 370,000

Dividends paid ( 380,000)


Proceeds from sale of treasury shares 750,000
Net cash provided by financing activities 370,000

11. What is the net cash used in investing activities? 1,700,000

Proceeds from sale of equipment 100,000


Purchase of bond investment (1,800,000)
Net cash provided in investing activities (1,700,000)

12. Jazz Company provided the following for the current year:

Increase in long-term debt 5,000,000


Purchase on treasury shares 1,000,000
Depreciation and amortization 1,500,000
Gain on sale of equipment 500,000
Proceeds from issuance of share capital 4,000,000
Purchase of equipment for cash 7,000,000
Proceeds from sale of equipment 2,000,000
Payment of cash dividend 2,500,000
Net income 8,000,000
Increase (decrease) in working capital account:
Accounts receivable 2,000,000
Inventory (3,500,000)
Trade accounts and notes payable 4,000,000
Income tax payable (4,500,000)
Cash balance, January 1 6,000,000

What is the net cash provided by operating activities? 10,000,000

Net income 8,000,000


Increase in account receivable (2,000,000)
Decrease in inventory 3,500,000
Increase in accounts and notes payable 4,000,000
Decrease in income tax payable (4,500,000)
Depreciation and amortization 1,500,000
Gain on sale of equipment ( 500,000)
Net cash provided – operating 10,000,000

13. What is the net cash used in investing activities? (5,000,000)

Purchase of equipment for cash (7,000,000)


Proceeds from sale of equipment 2,000,000
Net cash used – investing (5,000,000)

14. What is the net cash provided by financing activities? 5,500,000

Increase in long-term debt 5,000,000


Purchase of treasury shares (1,000,000)
Proceeds from issuance of share capital 4,000,000
Payment of cash dividend (2,500,000)
Net cash provided – financing 5,500,000

18. Zan Company provided the following data:

Trade accounts receivable, net 840,000 780,000


Inventory 1,500,000 1,400,000
Accounts payable 950,000 980,000

 Total sales were P12,000,000 for 2011 and P11,000 for 2010. Cash sales were
20% of total sales each year. Cost of goods sold was P8,400,000 for 2011.
 Variable general and administrative expenses for 2011 were P1,200,000. They
have varied in proportion to sales, 50% have been paid in the year incurred and
50% the following year. Unpaid G&A expenses are not included in accounts
payable.
 Fixed G&A expenses, including P350,000 depreciation and P50,000 bad debt,
totaled P1,000,000 each year. 80% of fixed G&A expenses involving cash were
paid in the year incurred and 20% the following year. Each year there was a
P50,000 bad debt estimate and a P50,000 writeoff. Unpaid G&A expenses are
not included in accounts payable.

What is the cash collected from customers during 2011? 12,010,000

Accounts receivable – 2010 `840,000


Sales – 2011 12,000,000
Total 12,840,000
Accounts receivable – 2011 ( 780,000)
Writeoff ( 50,000)
Cash collections in 2011 12,010,000

19. What cash disbursed during 2011 for purchases? 8,270,000

Accounts payable – 2010 950,000


Purchases 8,300,000
Total 9,250,000
Accounts payable – 2011 ( 980,000)
Payment of accounts payable in 2011 8,270,000

20 What is the cash disbursed during 2011 for expenses? 1,750,000

Fixed expenses 1,000,000


Depreciation ( 350,000)
Bad debt expense ( 50,000)
Fixed expenses paid in 2011 600,000
Variable expenses paid 2011:
2011 (1,200,000 x 50%) 600,000
2010 (1,100,000 x 50%) 550,000
Total cash disbursement for expenses 1,750,000
SINGLE ENTRY

1. On January 1, 2011, the capital of Console Company was P1,700,000 and on


December 31, 2011 the capital was P2,400,000. During the current year, Console
withdrew merchandise with carrying amount of P100,000 and sales value of P180,000,
and [aid a P1,000,000 note payable business with interEst of 12% for 6 months with a
check drown on a proposal checking account.

What is the net income or loss for 2011? P260,000 loss

Capital – December 31 P2,400,000


Add: Withdrawal – Merchandise @ carrying amount 100,000
Total P2,500,000
Less: Capital – January 1 1,700,000
Additional Investment 1,060,000 2.760,000
Net Loss P( 260,000)

The additional investment is determined as follows:


Payment of note payable out of personal checking account P1,000,000
Interest (1,000,000 x 12% x 6/12) 60,000
Total P1,060,000

2. The December 31, 2011 statement of financial position of Melisa Company showed
shareholder’s equity of P5,000,000. The share capital of P3,000,000 remained
unchanged during the year.

Transactions during the year which affected the equity were:

 An adjustment of retained earnings for 2010 overdepreciation P100,000


 Gain on sale of treasury shares 300,000
 Dividend declared, of which P400,000 was paid 600,000
 Net income for 2011 800,000

What is the balance of retained earnings on January 1, 2011? P1,400,000

Retained earnings – January 1,400,000


Add: Net Income 800,000
Prior period error of 2010
Overdepreciation 100,000 900,000
Total 2,300,000
Less: Dividend declared 600,000
Retained earnings – December 31 1,700,000

3. Audrey Company provided the following data:

12/31/10 12/31/11
Share capital (P100 par value) 5,000,000 5,500,000
Share premium 1,500,000 2,500,000
Retained earnings 3,000,000 4,500,000

During 2011, the entity declared and paid cash dividend of P1,000,000 and also
declared and issued a stock dividend. There were no other changes in equity during
2011.

What is the net income for 2011? P4,000,000

Increase in share capital (5,500,000 – 5,000,000) 500,000


Increase in share premium (2,500,000 – 1,500,000) 1,000,000
Stock dividend 1,500,000

Retained earnings – December 31, 2011 4,500,000


Stock dividend 1,500,000
Cash dividend 1,000,000
Total 7,000,000
Retained earnings – December 31, 2010 (3,000,000)
Net income 4,000,000

4. Sunshine Company had total assets of P4,000,000 and shareholders’ equity of


P2,080,000 at the beginning of the year. During the year, assets increased by P520,000
and liabilities decreased by P820,000.

What is the shareholders’ equity at the end of the year? P3,420,000

Effect on equity
Increase in assets 520,000
Decrease in liabilities 820,000
Net increase in equity 1,340,000
Shareholders’ equity – beginning 2,080,000
Shareholders’ equity – ending 3,420,000

5. Trend Company provided the following information for the current year:

Net loss 100,000


Total assets on December 31 3,000,000
Share capital on December 31 1,000,000
Share premium 500,000
Dividends declared 700,000

The debt-to-equity ratio (liabilitites over equity) is 50% on December 31. What is the
balance of retained earnings on January 1? 1,300,000

Shareholders’ equity (3,000,000 / 150%) 2,000,000


Less: Contributed capital (1,000,000 + 500,000) 1,500,000
Retained earnings – December 31 500,000

Retained earnings – January 1 1,300,000


Net loss ( 100,000)
Dividends declared ( 700,000)
Retained earnings – December 31 500,000

6. Easy Company’s beginning and ending total liabilities were P840,000 and P1,000,000,
respectively. At year-end, owners’ equity was P2,600,000 and total assets were
P200,000 larger than at the beginning of the year. If new share capital issed exceeded
dividends paid by P240,000, what is the net income (loss) for the year? 200,000
loss
Increase
Increase in asset 200,000
Increase in liabilities (1,000,000 – 840,000) (160,000)
Increase in owners’ equity 40,000
Excess of share capital issued over dividends paid (240,000)
Net loss (200,000)

7. The following changes in Vela Company’s account balances occurred during the current
year:

Increase
Assets 8,900,000
Liabilities 2,700,000
Share capital 6,000,000
Share premium 600,000

Except for a P1,300,000 dividend payment and the year’s earnings, there were no
changes in retained earnings for the year. What is the net income for the current
year? P900,000
Increase in assets 8,900,000
Increase in liabilities (2,700,000)
Net increase in equity 6,200,000
Add: dividend 1,300,000
Total 7,500,000
Less: Increase in share capital 6,000,000
Increase in share premium 600,000 6,600,000
Net income 900,000

8. An analysis of the records of Isabel Company disclosed changes in account balances


for the current year and the supplementary data listed below.

Cash 480,000 decrease


Accounts receivable 300,000 increase
Merchandise inventory 3,100,000 increase
Accounts payable 420,000 increase

During the year, Isabel borrowed P4,000,000 in notes from the bank and paid off notes
of P3,000,000 and interest of P240,000. Interest of P100,000 is accrued on December
31. There was no interest payable at the beginning of the year.

In the current year, Isabel transferred certain trading securities to the business and
these were sold for P1,500,000 to finance purchase of merchandise. Isabel made
weekly withdrawals in the current year of P10,000.

What is the net income for the current year? P420,000

Effect on equity
Increase Decrease
Decrease in cash 480,000
Increase in accounts receivable 300,000
Increase in inventory 3,100,000
Increase in accounts payable 420,000
Increase in notes payable (4,000,000 – 3,000,000) 1,000,000
Increase in accrued interest payable ________ 100,000
3,400,0002,000,000

Net increase in equity 1,400,000


Add: Withdrawals (10,000 x 52 weeks) 520,000
Total 1,920,000
Less: Additional investment (sale of securities) 1,500,000
Net income 420,000
9. Selected information for Marbel Company for the year follows:

Cash balance, January 1 130,000


Accounts receivable, January 1 190,000
Collections from customers 2,100,000
Shareholders’ equity, January 1 380,000
Total assets, January 1 750,000
Total assets, December 31 880,000
Cash balance, December 31 160,000
Accounts receivable, December 31 360,000
Total liabilities, December 31 390,000

What is the net income for the current year? 110,000

Total assets – December 31 880,000


Total liabilities – December 31 390,000
Shareholders’ equity – December 31 490,000
Shareholders’ equity – January 1 380,000
Net income 110,000

10. The changes in all the account balances of Mac Company for the current year, except
for retained earnings, are as follows:

Increase
(Decrease)
Cash 790,000
Accounts receivable, net 240,000
Inventory 1,270,000
Investments ( 470,000)
Accounts payable ( 380,000)
Bonds payable 820,000
Share capital 1,250,000
Share premium 130,000

There were no entries in the retained earnings account except for net income and a
dividend declaration of P190,000 which was paid in the current year.

What is the net income for the current year? 200,000

Increase in cash 790,000


Increase in accounts receivable 240,000
Increase in inventory 1,270,000
Decrease investments ( 470,000)
Decrease in accounts payable 380,000
Increase in bonds payable ( 820,000)
Net increase in equity 1,390,000
Add: Dividend declared 190,000
Total 1,580,000
Less: Increase in share capital 1,250,000
Increase in share premium 130,000 1,380,000
Net income 200,000

11. Daisy Santos started a retail merchandise business on January 1, 2011. During the
fiscal year ended December 31, 2011, the entity paid trade creditors P2,000,000 and
suffered a net loss of P350,000.

The ledger account pre-closing balances on December 31, 2011 include the following:

Accounts receivable 600,000


Accounts payable 750,000
Capital (total investment in cash) 2,000,000
Expenses (paid in cash) 100,000
Merchandise (unadjusted debit balance) 700,000

There were no withdrawals. All sales and purchases were on credit. The merchandise
account is debited for purchases and credited for sales.

What is the amount of sales for the year? 2,050,000

Accounts payable – December 31 750,000


Payments to trade creditors 2,000,000
Total purchases 2,750,000
Less: Unadjusted debit balance of merchandise account 700,000
Sales 2,050,000

12. What is the cash balance on December 31, 2011? 1,350,000

Cash – January 1 (investment) 2,000,000


Collections of AR (2,050,000 – 600,000) 1,450,000
Total 3,450,000
Less: Payment of account payable 2,000,000
Payment of expenses 100,000 2,100,000
Cash – December 31 1,350,000
13. What is the merchandise inventory on December 31, 2011? 450,000

Sales 2,050,000
Cost of Sales:
Purchases 2,750,000
Merchandise inventory – 12/31 ( 450,000) 2,300,000
Gross loss ( 250,000)
Expenses ( 100,000)
Net Loss ( 350,000)

14. Complex Company kept very limited records. On January 1, 2011, Complex Company
started business and issued share capital, 60,000 shares with P100 par, for the
following considerations:

Cash 500,000
Building (useful life, 15 years) 4,500,000
Land 1,500,000
6,500,000

An analysis of the bank statements showed total deposits, including the original cash
investment of P3,500,000. The balance in the bank statement on December 31, 2011,
was P250,000, out there were checks amounting to P50,000 dated in December but not
paid by the bank until January of next year. Cash on hand on December 31, 2011 was
P125,000 including customers’ deposit of P75,000.

During the year, Complex Company borrowed P500,000 from the bank and repaid
P125,000 and P25,000 interest. The proceeds of the loan were credited to the bank
account of Complex Company. Disbursements paid in cash during the year were as
follows:

Utilities 100,000
Salaries 100,000
Supplies 175,000
Taxes 25,000
Dividends 150,000
550,000

An inventory of merchandise taken on December 31, 2011 showed P755,000 of


merchandise. Tickets for accounts receivable totaled P900,000 but P50,000 of that
amount may prove uncollectable. Unpaid supplies invoices for merchandise amounted
to P350,000.
Equipment with a cash price of P400,000 was purchased in early January on a one-year
installment basis. During the year, checks for the downpayment and all maturing
installments totaled P445,000. The equipment has a useful life of 5 years.

What is the total cash on December 31, 2011? 325,000

Cash in bank per book 250,000


Outstanding checks ( 50,000)
Adjusted cash in bank 200,000
Cash on hand 125,000
Total cash – December 31, 2011 325,000

15. What is the amount of sales for the year? 4,000,000

Initial cash investment 500,000


Proceeds of loan 500,000
Collection of accounts receivable 2,500,000
Total deposits 3,500,000

Customers’ deposit 75,000


Collections of accounts receivable 600,000
Total 675,000
Disbursements in cash ( 550,000)
Cash on hand – December 31, 2011 125,000

Accounts receivable – December 31, 2011 900,000


Collections deposited 2,500,000
Collections not deposited 600,000
Total 4,000,000

16. What is the amount of purchases for the year? 3,055,000

Total deposits 3,500,000


Total disbursements in check (3,300,000)
Cash in bank – December 31, 2011 200,000

Payment of loan 125,000


Interest on loan 25,000
Payment for equipment 400,000
Interest on equipment 45,000
Payment of accounts payable 2,705,000
Total disbursements in check 3,300,000

Accounts payable – December 31, 2011 350,000


Payment of account payable 2,705,000
Total purchases 3,055,000

17. What is the net income for the year? 800,000

Sales 4,000,000
Cost of sales:
Purchases 3,055,000
Inventory – December 31, 2011 ( 755,000) 2,300,000
Gross income 1,700,000
Expenses:
Utilities 100,000
Salaries 100,000
Supplies 175,000
Taxes 25,000
Doubtful accounts 50,000
Depreciation – building (4,500,000/15) 300,000
Depreciation – equipment (400,000/5) 80,000
Interest expense (25,000 + 45,000) 70,000 900,000
Net income 800,000

18. What is the amount of total assets on December 31, 2011? 7,950,000

Cash 325,000
Accounts receivable 900,000
Allowance for doubtful accounts ( 50,000)
Inventory 755,000
Land 1,500,000
Building 4,500,000
Accumulated depreciation – building ( 300,000)
Equipment 400,000
Accumulated depreciation – equipment ( 80,000)
Total assets 7,950,000

Liabilities 800,000
Shareholders’ equity 7,150,000
Total liabilities and shareholders’ equity 7,950,000

19. What is the amount of total liabilities on December 31, 2011? 800,000

Accounts payable 350,000


Loans payable – bank (500,000 – 125,000) 375,000
Customers’ deposit 75,000
Total liabilities 800,000

20. What is the amount of shareholders’ equity on December 31, 2011? 7,150,000

Share capital (60,000 x P100) 6,000,000


Share premium (6,500,000 – 6,000,000) 500,000
Retained earnings 650,000
Total shareholders’ equity 7,150,000

Net income 800,000


Dividends paid ( 150,000)
Retained earnings 650,000

S-ar putea să vă placă și